Chapte: Charges
Chapte: Charges
                                                                                                                   Elecric                                   Charges
                                                                                                                                                      and                        Field
1.1      FRICTIONAL                    ELECTRICITY                                                           electrica   for such substan ces. In fact, the Greek                              name for
                                                                                                             amber is elektron             which      is   the origin of all such               words :
   1. What is frictional electricity                            ? When        is   a body          said      electricity,      electric      force,     electric      charge          and electron.
tobe electrified or charged ?
      Frictional       electricity.              If    aglass    rod   is   rubbed with a                                             For Your Knowledge
silk cloth,      or a fountain-pen with a                          coat-sleeve,          it   is   able
to attract     small pieces of paper, straw, lint, light                                feathers,
                                                                                                                     Amber       is    a   yellow      resinous       (gum like)           substance
                                                                                                                     found on the shores              of   the Baltic sea.
etc.   Similarly, a plastic                      comb     passed through dry hair
can    attract       such light            objects.      In    all   these examples,                we               Both electric and magneticphenomena can be derived
                                                                                                                     from       charged        particles.          Magnetism            arises     from
can say that the rubbed substance has                                  becomeelectrified
                                                                                                                     charges      in       motion.   The charged            particles      in    motion
or    electrically      charged.            It   is   because of        friction         that       the
                                                                                                                     exert   both electric and magnetic forces on each                             other.
substances get charged                           on    rubbing.
                                                                                                                     Hence      electricity     and magnetism are                studied        together
       The    property      of rubbed                 substances       due to which                 they
                                                                                                                     as electromnagnetism.
 attract     light     objects        is     called      electricity.         The       electricity
 electrically         charged.
                                                                                                                    Electric     charge. Electric charge is an                       intrinsic    property
       2. Give       a historical           view of frictional           electricity.           From          of the elementary particles                      like          protons, etc.,
                                                                                                                                                                      electrons,
 where did the term               electricity             get    its origin        ?                          of which all the objects are                    made up of. It is because of
       Historical view of                    frictional        electricity.        In   600        B.C.,      these electric           charges        that   various objects exert strong
                                                                                                              electric    forces       of attraction         or repulsion on each other.
 Thales ofMiletus,             one of the               founders of Greek                 science,
                                          personal doctor to
     In 1600 A.D., William Gillbert, the                                                                                                                                (+ e) and an
                                                                                                               conlomb (C).A               proton has a positive charge
                  -Iof England,    made   a systematic study                                                                                                (-è), where
 Queen Elizabeth                                                                                                        has a              negative charge
                                         amber.  In his book                                                   electron
                         behave     like
 of the substances that
                                                       name                                                                            e=1.6x 10                coulomb
 De Magnete (on the magnet), he introduced the
                                                                                                           (1.1)
                                                             of egal        uberod                                                                                              rt
         Large-scale matter
                                that coests
                                                                  tal       M hene       s                rgedes                    od         The     t
   electrons and
                          peotos is ectrially                                 harge
                            bdy hasa
         excsof clectr, theeslts
                                                                    negative
    an
                                 ia potie                                    darge
   and an exoese          of   proons
                                                                                                          fros     apd upg                          Being       eer                       ey
                                                                                                                                                                                          be
                                                                                                          harped                               d       r            Thene           l
   13     ELECTROSTATICS                                                                                                  plastic
 important applicats
                                                                                                    Easr               2.   Ma      asd.                            bed     wi
     Electrostatics Electrstatics                    s he stuy f               ciecrie         made      o foxct teo sml pih bls  peirtye
                                                                                                                              eadsen
                                                                                                                                                                    for
                                                                                                                                              Bs
 repulsion between charged bodies
                        Some of these are as follows:     seen that a pih   ball toched             nd                                 wi ss rats
 trial applications
                                                          anoherpi bull toached    wita  plaic  od JFi                                                                           123
      1. In electrostatic  loudspeaker.
                                               and powder
     2. In electrostatic spraying of paints
          coating
     3. In flyash collection in                  chineys
     4. In       aXerox      copying          machine.
                                                                                                                                         oached                      agas od as
                                                                                                                                                                                 e hurge
                                                                                                The statement
                                                                                            of electrostatics
                                                                                                                                 demcsrate hut te
                                                                                                                                             also
                                           PRastic                                              The aboeexperimenits                             bas
                                                                                                               traserned from the ods
                                                                                                                                      o  e pitth
                                                                                              harges
                                                                                         on contact
                                                                                                         are
                                                                                                         tekonds
                                                                                            dotinguihes te                               crges                  calod     the    polerity
                                                     le                                                                                  f
                                                                                                                                                           is
         Fig   1.1    Lâe chasges epel and                         chages                ofcharge
                             attract   each othe.
                                                                              ELECTRIC       CHARGES AND                    FIELD                                                                           1.3
6. What are vitreous and resinoUS charges What turns out to be negative in this convention. It would
                                                                                         ?
     was wrong with             this   nomenclature?                                                             haye been more                     convenient      if   electrons        were       assigned
    Vitreous and resinous charges,Charles Du Fay used the                                                        positive         charge.       But inscience,             sometimes            we have to
                                                                                                                 live with          the      historical      conventions.
 terms vitreous and resinous for the two kinds of charges.
1. The charge developed on gass rod when rubbed with Different substances can be arranged in a series in
            silk ons called vitreous                                                                             Such        away that         if   any two of them arerubbed together,
                                     charge (Latin virlum =glass).
                                                                                                                 then       the    one    occurring          earlier in        theseries         acquires       a
      2.    The charge developed on amber when rubbed with
                                                                                                                 positive         charge while            the    other occurring              later acquires
            wool was called resinous charge (amberis a resin).
                                                                                                                 a    negative       charge
       But     later       on, these
                                 were found to be   terms
                                                                                                                 1.         Fur                2.       Flannel                    3.          Sealing    wax
 misleading. For example, a ground glass rod develops
                                                                                                                 4.         Glass              5.       Cotton                     6.          Paper
 resinous       electricity           while     a   highly polished ebonite rod
                                                                                                                 7.         Silk               8.       Human body                  9          Wood
 develops vitreous                electricity.
                                                                                                               10.          Metals            11.       Rubber                      12         Resin
       7.   What are positive                 and    negative        charges          ? What       is
                                                                                                                            Amber                                                  15.         Ebonite
 the nature                                                                                                      13.                          14.      Sulphur
                  of    charge on an             electron      in    this     convention           ?
                                                                                                               16.          Guta     parcha
       Positive        and      negative charges. Benjamin Franklin
 (1706-1790), an                American pioneer of electrostatics                                                                                     a    positive       charge when                 rubbed
                                                                                                              Thus          glass    acquires
introduced the present-day convention by replacing                                                            with      silk      but   it   acquires        negative       charge when rubbed
the terms vitreous and resinous by positive and                                                               with
                                                                                                   :
                                                                                                                        flarnnel.
                                                     Amber      rod                                     when a glass rod is rubbed with a silk cloth, some
        Woollen         cloth
                                                                                                        electrons are transferred from glass rod to                                                    silk.    The
        Woollen         coat                         Plastic   seat
                                                                                                        glass rod develops                      a positive charge                   due         to   deficiency
                                                     Rubber shoes
        Woollen         carpet                                                                          of electrons while                          the     silk    cloth develops                     an     equal
two different columns will attract each other. stillzero, as it was before rubbing i.e., electric charge is
                                                            and negative charges                        which an electron can be pulled away from an atom is
        Benjamine'schoice of                  positive                                        is
       purely      conventional            one.     However,          it is   unfortunate               to exert a strong                      electric          on it. As electrons are
                                                                                                                                                                 force
       that the charge on an electron                       (which     is     so important              actually transferred from                           one body to another during
       to    physical      and         chemical        properties           of   materials)             rubbing, so            frictional            forces must have an electric origin.
                                                                                               PHYSICS          X
                                                                                                                                              EA                                         B
this distinguishing                 feature in the next chapter.                                                                               Excess         of                   Deficiency      of
                                                                                                                             charged
    A   metal rod held in                   hand and             rubbed         with wool                                     rod
                                                                                                                                               electrons                           electrons
                                                                                                                 Positively
does not develop any charge. This is because the                                                                       glass
household circuits ? What is its importance ? temporary electrification of a conductor in which opposite
Earthing and safety. When a charged body is charges appear at its closer end and similar charges aypear at
brought     in   contact with the earth (through a connecting                                          its   farther end in the presence                 of   a     nearby charged body.
conductor),         its   entire charge passes                    to    the   ground in                       The positive and           negative charges produced                                 at   the
the form of a momentary current. This process                                   in      which a        ends of      the conducting rod are called                         induced charges               and
body shares      its   charges with the            earth   is called      grounding or                 the charge       on the glass                    rod         which          induces these
earthing.                                                                                              charges      on conducting rod                   is called         inducing charge.
                                                                          ELECTRIC          CHARGES AND               FIELD                                                                        15
         12.      Describe         hoe tuometal splheres                can beopposilely                 13.       How     can you charge                        a   metal sphere         positiuely
  carged by             induction.                                                                 wilthout         touching         it    ?
         Charging             of two spheres by induction. Figure 1.5                                 Charging of a sphere by induction. Fig,                                          16     shows
  shows the various steps involved              in inducing,opposite                               the  various steps involved in inducing                                            a     positive
  charges on two metal spheres.                                                                    charge on a metal sphere.
                                                                                                         (a)    Hold  the metal sphere on an insulating     stand.
                                                                                                                Bring a negatively charged plastic rod near
                                                                                                                                                                                                        it.
                                                                                                                The free electrons of the sphere are repelled to
                                                                                                                the farther               end. The near end becomes posi
                                                                                                                tively     charged due           to deficit of electrons.
(e)
Plastic
Fig. 1.5 Two metal spheres get oppositely charged by induction. rod
begins to build up at the left                      face,   it   starts repelling the                    Similarly,        the metal sphere can be negatively charged
new   incoming   electrons.  Soon an equilibrium is                                                by bringing a positively charged glass rod near                                                it.
                                                                                                           of      divergence        of        the
      Thus           the    two       metal      spheres         get charged            by    a
                                                                                                            leaves                               a                                         Gold
                                                                                                                           gives                                                                   leaf
process        called        charging by         induction.        In   contrast       to   the
                                                                                                            measure             of             the
process         of   charging         by contact,here            the glass rod does                                                                                                        Tin   foil
                                                                                                           amount         of charge.
not lose any of               its    charge.
     I:6                                                                                          PHYSICS-XI|
18 BASIC PROPERTIES OF ELECTRIC CHARGE The cxperimental fact that electric charges occur in
      Additivity                       of   electriccharge means that the otal                           charge ? Ifyes, under what conditions ?
 charge ofa system                       is the algebraic sum ofall the individual                           When can we ignore the quantization of electric
                                                                the system.
 charges located at                      different pointsinside
                                                                                                         charge.While dealing with macroscopic charges (9 = ne),
         Ifa     system                contains charges              4,,4,-.,qpr           then   its
                                                                                                             can ignore the quantization of electric
                                                                                                                                                              charge.   This
                                                                                                        we
                                                                                                                                               very  small  and    n is very
 total    charge             is                                                                          is because the basiccharge e is
                                                                                                                                                   so  q behaves as if it
                                                                                                         large in most practical     situations,
quantily             ?
                                                                         quanti                                              For Your Knowledge
     Quantization                       of a  physical quantity. The
                                                                      vary conti                                                                          charge or basic           quantum        of
                                       quantity imeans that it cannot                                         The     smallest     amount           of
 Units Used
           Mass
           qand
                  transferred
                   e are in coulomb, n
                                            during charging
                                                          is   pure
                                                                            = m, x
                                                                           integer.
                                                                                      n
                                                                                                          .
                                                                                                       of water
                                                                                                               Number
                                                                                                                                     =   Avogadro's
                                                                                                                                of molecules present
                                                                                                                                                                  number = 6.02
                                                                                                                                                                           in    a cup (or 250 g)
                                                                                                                                                                                                   1023
 Constants         Used
                                                                                                                                n= 6.02
                                                                                                                                                 x   10x 250 =8.36                       x 1024
          e= 1.6       x   10C, m, =9.1                    x   10          kg
                                                                                                                                                       18
Example 2.A                comb drawn through person's                           hair     on a dry       Problems For Practice
day causes       10        electrons           to leave   the person's           hair     and stick
to the comb. Calculate                    the    charge carried by the comb.                              1.   Calculate         the charge carried                 by     12.5       x    10   electrons.
Charge given out by the body in one second 5. Determine the total charge on 75.0 kg of electrons.
                                         6.25x 109
                                                                                                                 =   18.                                                         (Ans. 5.35 x 10° C)
                                                                years =198.18 years.
                                 365 x 24x 3600                                                        HINTS
   Thus from                    a   body          emitting           10         electrons per
                                                                                                          3. An            alpha           particle           contains               2      protons            and
second, it        will     take nearly 200 years                       to    get a charge of                     2 neutrons.
1C from           that      body. This shows how large                                      is   one
                                                                                                                                         q=2e.
coulomb as         the unit of charge.
                                                                                                                           PHYSICS- XIl
Fe nucleus contains 26 protons and 30 neutrons. 4. Electric charge is conserved during the
5. H
Law of conservation of charge. If some amount of nature. It is valid in all domains of nature. Even in the
matter is isolated in a certain region of space and no                                                                                     domains           of       physics, where mass changes
                                                                                                                                                                  high energy
               enters or leaves this region by moving
                                                                                                                                                                                                  of
matter either                                                                                                                                           and               the law of conservation
                                                                                                                                           into energy      vice-versa,
across      its   boundary,                then whatever                                other changes                  may                 charge strictly holds good.
                                                                                    total         charge        will       not
occur       in    the matter                   inside,                      its
change with time. This                                  is       the          law of conservation of                              112 ELECTRIC CHARGE VS MASS
charge which states                                                                                                                          Compare the properties                                  of    electric        charge with
                                          :
                                                                                                                                         19.
the entire universe remains constant. positive, negative or zero. always positive.
                                                                                                                                                                                                    Gravitational           forces
   3. Charge              is    conserved during the                                          fission        of a                  6.      Electrostatic            forces
                                                                                                                                                                                                                                     bodies
                                                                                                                                          between            different                             between different
            nucleus by                a   neutron.                                                                                                                                                 never cancel out.
                                                                                                                                          charges may               cancel          out.
                                                                                                                                                                    1
to the      special            theory         of    relativity,         the    mass of a body
increases with                 its   speed          in   accodance            with the            relation:
                                                                                                                                                                     1
                                                                                                                                                        F=                      -9x10"                   N
In contrast to mass, the charge on a body remains (ii) In electrostatic cgs system, the unit of charge is
 constant and does not change as the speed of the                                                        body known          as electrostatic                             unit          ofcharge (e.s.u. of charge) or
 changes.                                                                                                             statcoulomb (stat C).
                                                                                                                                                                              or one statcoulomb                                        charge
                                                                                                                            One e.s.l. of charge                                                                           is   that
21. State Coulomb's law in electrostatics. Express the one centimetre from it
same in SI units. Name and define the units of electric l coulomb =3x 10 statcoulomb
Coulomb's law states that the force of attraction or A torsion balance is a sensitive device to measure force.
 proportional    to the product of the magnitudes                                                 of    the   two                  than       the distance         between them,                     their sizes         may be ignored
 charges       and           (ii) inversely              proportional          to       the   square of       the
                                                                                                                                   and        the   charged bodies are                             called      point      charges.
 distance between them. This force acts along                                               the line joining
                                                                                                                                    Coulomb's law is                         valid        only      for      point charges.
 the two charges.
                                                                                                                                   In     SI   units,     the exact value of the combination 4n                                          s,is
                                                                                                                                                                                   10
                                                                                                                                                                                          CN'm²
                                         Fig. 1.7         Coulomb's      law.
      114 COULOMBS LAW                                             IN VECTOR                    FORM                                 Rangeof coulornbian forces.Coulombian forces act
                                                                                                                                                                                                                                     The ratio (E/E,)of tM
                                                                                                                                  over  an enormous rarnge   of  separationis (r, fronm
             22. Write               Coulomnb's law                       vector form. What                                                                 15
                                                                                                                                                                                                                                permittiity (E,)                 of    fre
                                                                   in                                               is    the     nucear dimensions (r= 10      m)to macroscopic dis
     importance                     expressing            it    in    ector form                ?                                                                                                                                (6,jor     delectric           costa
                                                                                                                                  tances as large as 10                m. Inverse                square                  over
                               of
                                                                                                                                                                                                             is valid
        Coulomb's law in vector form.As shown in Fig. L8,                                                                         this     range of separation         to          a high degree             of accuraey
     considertwo positive point charges q, and q, placed in                                                                           Limitations of Coulomb's law. Coulomb's law is
     vacuum at distance r from each other. They repel cach                                                                        not applicable in all situations.                                                                                can defin
     other.
                                                                                                                                                                    It is valid only under                                           So one
                                                                                                                                  the following conditions                :                                                     forces between charg
                                                                                                                                         1.    The electric   charges must be                      at rest                           The     dielectric               con
                                                                                                                                                                                                                                vacuum.That is,
                                              1
                                                                                                                                 1.15        DIELECTRIC CONSTANT:
                                                                                                                                             RELATIVE PERMITTIVITY
                                                                                                                                                                                                         ofa     medium?           Hence the Coulon
  where                  =2,         is a    unit vector             in    the direction from                            4,           24.     What do you mean         by          permittivity
                                                                                                                                                                                            of
 to    G2
                                                                                                                                charges.
       Similarly,                     Fp    =Force on charge                          4,   due      to    4,                        Permittivity :An introduction. When two charges
                                                                                                                                                                                    the force
                                                                                                                                are placed in any medium other than air,                                                                              KvacuL
                                                    1
                                                                   2                                                            between
                                                                                                                                         of
                                                                                                                                           them
                                                                                                                                            the
                                                                                                                                                  is greatly
                                                                                                                                                medium   which
                                                                                                                                                               affected.
                                                                                                                                                                determines
                                                                                                                                                                          Permittivity    is a
 where f,                =21,is a unit vector                        in   the direction from 4,                                 the force between two charges located some distance                                             Examples based
                                                                                                                                      in water is about 1/80th of the force between
                                                                                                                                apart
                                                                                                                                                                                                                                  Coulomb's Law
to                                                                                                                              them when they are separated by same distance
      41                                                                                                                                                                      in air.
      The coulombian forces between                                                unlike charges                              This     is    because      the absolute            permittivity             of   water    is     Formulae Used
                                                                                                                                                                                                                         of
(9,4, <0) are              attractive,            as    shown             in   Fig.      1.9.
                                                                                                                               about 80 times greater than the absolute
                                                                                                                                                                                                       permittivity                                     1
                               +41           F2
                                                                            -42                                                ding   to      Coulomb's law, the force between two point                                            2.    ed
                                                                                                                               charges 9,       and g,, placed in vacuumn at distance r trom
                                                                                                                                                                                                                                 Units Used
                                                                                                                               each other, is given by
           Fig. 1.9        Attractive         coulombian             forces       for   g, 4,   < 0.                                                                           1
                                                                                                                                                                                                                       ..1)               3are in cou
            Thismeans that                    the    two charges                  exert equal                 and                                                                                                      .2)      Example 5. The electr
            opposite                forces     on       each other.                So      Coulombian                                                       Fmed      4TE                                                                     charged           ions         C
                                                                                                                                                                                                                                positiely
                                                                                                                                                                                                                                     Solution. Here                     F
            Le, along                the     line    joining the centres                            of    two                 equation (1)by equation              (2),     we       get
            charges, so they are                                                                                                                                                                                                                        r=5A
                                                                                                                                                                           1
                                                        central       forces.
                                                                                                                                                                                                                                                                         1
      23.      What       is   the range over which Coulombian                                       forces                                                Fyac
                                                                                                                                                                                                                                     As                 F=
 can act             ?   State      the     limitations              of Coulomb's                   law        in
                                                                                                                                                            med
                                                                                                                                                                            1       412                                                                               4T
electrostatics.
                                                                                                                                                                      41cE
                                                                                                             ELECTRIC             CHARGES AND                 FIELD                                                                           111
forces act
(r),     from               The ratio (s/    c)ofthe permittivity                                    (s)of the medium           to the
                                                                                                                                                                              9x 10 x qx4
                                                                                                                                               ..      3.7x 10-9
opic        dis       permittivity        (E,)                  free    space     is       called relative            permitlivity                                                (5x 1010
                                                       of
walid    over          (E,)or    dielectric            constant (K)of the                                  mediun. Thus
accuracy.
                                                                                                 given
                                                                                                                                         or
                                                                                                                                                                                  3.7    x   10x25x 10-20                        =10.28x        1038
                                                                                            Fvat
b's   law       is                             E, Or                   K=                                                                                                                     9x           10
nly    under
                                                                                           Fmed
                                                                                                                                         or                              q=3.2 x 10-19                  c
                         So one can define dielectric constant                                                        in   terms    of
                                                                                                                                               Number               of electrons missing from each ion                                        is
                      forces betweencharges as follows :
                                                                                                                                                                                         3.2 x        10-19
                         The dielectric constant or relative permittivity ofa                                                                                                                                         2.
arges i.e.,                                                                                                                                                                                           10-19
                      medium may be defined as the ratio of the force between two                                                                                                        L.6x
 h   smaller
ges.                   charges placedsome distance                                apart in free space to the force                       Example 6. A                     free          pith-bal           A    of   8g        carries    a   positive
must be betoeen the same two charges cohen theyare placed the same charge of 5x 108 c What must be the nature and
, because
lear force
                      distance apart in the given medium.
                                                                                                                                                                                                  the
                                                                                                                                                                                                                     be given
                                                                                                                                                                                                           former ball
                                                                                                                                                                                                                                      to a second
                       constant K          is      placed between                               the charges, the force                   ball     is   stationary             ?                                                       (Haryana 01]
                       betweenthem becomes                                  1   / K times the original                       force in           Solution. The pith-ball Bmust be of positivecharge
                       vacuum. That is,                                                                                                   i.e.,  of same nature as that of A, so that the upward
                                                                                  Fvac
                                                                                                                                         force of repulsion balances the weight of pith-ball                                                         A.
                                                                   Fmed      =         K
                                                                                                                                                  When             the   pith-ball                Aremains
anedium?
                          Hence the Coulomb's                                   law for any material medium                              stationary,
ween two
                       may be written as
                                                                                                                                                                         F= mm8                                                                      91
o charges
the force
                                                           =       Fmed
                                                                                           1
                                                                                                        2                                                1 912
                                                                                                                                                               Mh8           =                                                     5cm
ivity      is     a                            K   (vacuum)=                      1
ectric
 example,
         force
                                                       K (water)
                                                                   K   (air) =1.00054
                                                                                = 80.
                                                                                                                                                  But                    m =8g=8
                                                                                                                                                                         9,   =5x 1o-
                                                                                                                                                                                                  x   10
                                                                                                                                                                                                      c
                                                                                                                                                                                                                 kg
                                                                                                                                                                                                                                          B)          42
e distance
                        Examples based on                                                                                                                                 r=5 cmn             =0.05            nm                        Fig. 1.10
 between
nce in air.              Coulomb's Law                                                                                                                  9x         10    x   5x       10          x   42   _8x10- x9.8
 water          is
                         Formulae Used                                                                                                                                   (0.05)²
ittivity        of
                                                   1
                                                                                                                                                                                  8x9.8x (0.05)                       x 10-4
                           1.    Fac
                                                                       2                                                                  or
                                                                                                                                                                                                       9x 5
ky.    Accor
                                                            1
                         Units Used
                                                                                                                                          Example 7.                      Aparticle          of mass            m    and      carrying     charge    -     q
..(1) 44 are in coulomb, F in newton and r in metre. radius r. Prove that the period of revolution of the charge -4,
                                                                                                                           betweentwo
          .(2)            Example         5.    The electrostaticforce                               of repulsion
                                                                                                                 is   3.7 x 10   N,
                                                                                                                                                    Solution.                Suppose charge                          - 4,     moves       around          the
                                         charged                ions    carrying       equal          charges
                                                                                                                                                              +4
                         positively
                                                                                                                                                                         with speed                   v along the                 circular      path       of
                          when    they     are separated                    by a           distance       of    5 A.       How many        charge
ity   or just                                                                                                                              radius             r.   Then
                         electrons     are         missing              ?
                                                                       from each               ion
 Dividing
                                                                        N,                                                                          Force of             attraction           between                 the two charges
                                Solution.              Here F =3.7 x 10
                                                                                                                                                                                  =   Centripetal force
                                                       r=5Å=5x 10 m,                                            =4, =9(say)
                             As                        F=               1
                                                                                9192
                                                                                                           4,
                                                                                                                                                               1         912             mu
                                                                                                                                                                                                           Or
                                                                                                                                                                                                                                  V4nE, mr
                                                                                                                                                                                                                                              9142
                                                                   4E              r
   112                                                                                                     PHYSICS-XI
         The period of revolution of charge                                   -   4,   around              +q,               (b) ()When charge on each sphere is doubled, and
  will    be                                                                                                              the distance between them is halved, the force of
                                                                                                                                                                                                                                                      Solution. Let the charge
          T= 2r -2r                                                           16n',mr3                                    repulsion becomes
                                                           Or      T                                                                                                                                                                               and B be    q.   Ifthe separation
                                                                                                                                                           24,24;
                                                                                         9192
                                                                                                                                                        =k."                          16    k 12                                                   electrostatic    force         betweern            sp
                                                                                                                                                            (r/2'
  Example8. Tuo                                                                                                                                                                                                                                                           F   =k.                     20
 mass
                                 identical
           10g and charge 2.0 x 10Care placed
                                                   charged particles
                                                                                   on a
                                                                                       each     having a
                                                                                              horizontal
                                                                                                                                                            -16x 1.5x 10                   =0.24 N.                                                                                      2
 table    with   a separation               L betwcen them                                                                  (i) The force between two                                         charges placed                 in    a                  When sphere C is touch
                                                                        such           that    they        stay           medium of dielectric constant k
 in limited      equilibrium.               If the coefficient
                                                                                                                                                                                             is    given by                                        charge q/2 each, because b
                                                                                  friction      between
                                                                         of
                                                                                                                                                                   1
 each    particle     and       the                                                                                                                     F=                       1   9142
                                                                                                                                                                                                                                                              Force on C due                   to
                                       table is 0.25,           find   the        value of
                                                                                                                                                                                      2
                                                                                                                                                                                                                                                                                                           A
                                                                                                      L.
                                                                                                                                                        third
                                                                                                                                                                             spheres
                                                                                                                                                                   sphere of the
                                                                                                                                                                                            A     and B   in
Example 9. (a) Two insulated charged copper spheres A Contact with sphere A, each sphere shares a charge charges are in equilibriun
and B have their centres separated by a distance    50cm.                                                                      /2,      equally.                                                                                                       posifion      of the         charge q
                                                                                                 of
What is the mutual force of electrostatic repulsion if the                                                                                                      Charge=0                               4/2       9/2                                                  Suppose
                                                                                                                                                                                                                                                              Solution.
charge on each             is    6.5    x    10C? The                  radii        of     A   and         B are                                    A                                                                                                   the   manner,as shown
negligible       compared              to        the    distance        of        separation.                  Also
                                                                                                                                                                           4/2                         34/434/4
compare this force with                  their         mutual     gravitational                 attraction
if each weighs 0.5 kg.
                                                                                                                                                                                                          B
                                                                                                                                                                           9/2                         34/4
    (0) What          is    the force of repulsion                      if (0) each              sphere              is
                                                                                                                               placed at the midpoint between                                     A   and        B.Calculate the net                            Example 13. Tw
    Clearly,          F     <<        Fuir
                                                                                                                               electrostatic            force on C.                                                    (CBSE OD 03]                             distance 'a'              apart.
    When                                                                                                                  + 4e
                     sphere C     is touched to                         A, the     spheres share
charge     q/2            each, because both are                        identical.
    ..     Force           on C due             to   A                                                                                                                                              a-X
                                     =k.(q/2)²                                                  AC
                                               (r/2)               2'           along                                                                                Fig. 1.13
            A                                                                                  -C                  equilibrium
                                                                                                                   increase and a net force (E,
                                                                                                                                                   position,      then          F, will
                                                                                                                                                                                   -E)
                                                                                                                                                                                             decrease and F,
                                                                                                                                                                                               will act    on g towards
                                                                                                                                                                                                                           will
       The   charge                 q    will      be in equilibrium                      if    the forces                Now        if   we take charge                       q to    be negative, the            forces      E
exerted      on           it   by    the charges at                A    and      C   are equal and                 and E, will be                        attractive,       as      shown        in   Fig.    1.14.
opposite.
                                                                                                                            +4e                                                                                         +e
                     k                                             Or            x=(r-x'
                                            (r-x?                                                                                                                          F                          F,
X=r-X Or -a-x
                                                                                                                                                                          Fig. 1.14
       Since the charge at                           A   is    repelled by the                       similar
                     Q.4 =k Q.Q
                                                                                                                    Example 14 Two                              free'       point charges                + 4e     and   + e are
                  (r/2'                                                                                            placed        a   distance              'a' apart.      Where should a                third point     charge
                                                                                                                                                                                                            system      mnay     be
Example 13.Two point charges + 4e and + e are fixed' a q be placed between them such that
                                                                                                                                                                                            the entire
                                                                                                                                                                                                                          ofq
           a'apart. Where should a third point charge g be in equilibrium ? What should
                                                                                          be the magnitude and sign                                                                                                                 ?
 distance
                                                                 What      an              will it be ?
                            the two charges so that it may be in      type    equilibrium
 placed on the line joining
                                                                                                                                              of
I14                                                                                               PHYSICS
                                                                                                                  XI
          Solution.          Supposethe charges are                               placed as shown           Example     16. A charge Qie to be divnded on tuo ohjects
                                                                                                             What shoud be the vales of the charges on the tuo objects
                                                                                                                                                                                                                          Now           (4,   +4,=(4, -42)
in        Fig.    1.15.
                                                                                                                                                                                                                                                         - (2 x 10
                        44e                                                                                 so that the force between the objects can be maximum
                                                                                                                                                                                                              ?
                                                                                                                                                                                                                                                         - 16x 10
                                                                                                                    Solution. Let                 q   and        Q-qbe            the charges on the two
                                                                                                                                                          1
                                                                                                                                          F=                                                                                                                          and
                                                                                                                                                                                                                                    4         3    10        C
                                                  Fig. 1.15
                                                                                                                                                                                                                        which are the              initial       charge
                                                                                                            where r is             the distance between                            the   two objects.
          As the        charge + e exerts repulsion F                                on charge + 4c,
                                                                                                                       For F   to    be maximum,                                                                        Example 18. Tuo small sp
so for the                 equililbrium           of   charge                + 4e,the charge - 4                                                                                                                        and charge q coulamb are
must             exert       attraction           F on +4e.                   This requires the                                                                                                                                                         each l metr
                                                                                                                                                                                                                        insulating        threads
charge q                   be                                                                                                                                          dq
                      to         negative.                                                                                                                                                                              9 is   the      angle,      euch     thread
                                    1     4exe                                                                                                        d
                                                            1           4exq                                 or                                               (9Q-g)=0                                                         Solution.   The given sit
                                                       4TtE0                                                                                      dq
                                                                              x                                                                                                                                          Each       of the spheres A a
                                                           ex
                                                                                                             Or                                                  Q-2q =0                                                 following forces
Or
                                                                                                             or                                                                                                                         () its      weight mg,
                                                                                                                                                                                   2
          For equilibrium of charge
                                                                                                                                                                                                                                     () the force                of rep
                                                                -4                                           i.e,       the charge should                        be divided equally on the two                                                                    1
          Attraction                F,   between+4eand                       -q                              objects.
                                                                                                                                                                                                                                                     F=
                                          =Attraction E, between                       + e and    -q         Example                 17.      Two             identical  spheres,  having charges of
                  1         4ex                        exq                                                                                                        other with a force of 0. 108 Nwhen
                                             1
                                                                                                             opposite         sign    attract             each
                             x                                                                               separated          by    0.5       m. The           spheres      are connected by a             conduc
or x'=4 (a -x ting wire, which then removed,and thereafter they repel each
                                                                                                                        (b)When                           two
     Q,
                                                    [CBSE D 98]                                                                               the                  spheres are connected                      by the
                                                                                                                                                                                                                             sphere A can be rept
                                                                                                              wire, they share the charges equally.                                                                          A AOCtaken in the sa
                                Suppose the                three charges are placed as
shown
           Solution.
                   in       Fig. 1.16.                                                                                  :     Charge on each sphere                               = 91+(-42)_1-92
                                                                                                                                                                                       2       2
                                                                                                                                                                                                                                                         F         mg
                  4                                                                                                                                                                                                                                     AC         OC
                                                                                                                                                                                    them
                  A                                                                           B                         Force of repulsion between                                           is
                                                                                                                                                                                                                               Or                            F   =       g
                                                                                                                                                           k
                                                    Fig. 1.16
                                                                                                                                                F=                                                                                      From ()and                (in), v
As 0.036 x (0.5) x4 1
                                1 4qx9                          1
                                                                                                                                                                                           =4x 1012
                                                                                                                                                                     9x      10                                                                    4nE       4I       sin
                                                                         (x/2'
                            4nE, (x/2)'                     47
                                                                                                                                                                                                                                Or
                                                  Q= 44.                                                                             41   -9 =2 x 10-6
                                                                                  ELECTRIC        CHARGES AND                  FIELD                                                                      1.15
Now
    On
                 (4,+t 9,)'
         solving equations
                            t95
                                     -(4,-4,) + 494:
                                     -(2 x 10+4x3x
                                      =
                                      =
                                          16x
                                          4x 10 6
                                                   10
                                                   (i)
                                                         12
                                                         and        (ii),   we
                                                                                  10
                                                                                  get
                                                                                        12
                                                                                                  ...i)
                                                                                                               1.
                                                                                                               2.
                                                                                                                 Problems For
                                                                                                                     Obtain the dinensional formula
                                                                                                                     Calculate coulomb
                                                                                                                                                        Practice
                                                                                                                                                               force
                                                                                                                                                                                     of
                                                                                                                                                                                  (Ans.
                                                                                                                                                                           between two
                                                                                                                                                                       of 3.2 x      101 m
                                                                                                                                                                                          :     M    'L TA?)
                                                                                                                                                                                                    u-particles
                                                                                                                                                                                                    in    air.
                                                                                                                      separated             by      a distance
         4,   =3x           10       C         and                    =106C
                                                              9,                                                                                                                                [CBSE      OD         92)
which are the                             charges on the two spheres.
                           initial                                                                                                                                                                  (Ans. 90          N)
Example 18. Tw0                          having nass m kg
                                     small         spheres each                                                3.     Calculate             the      distance         between         two protons such
and charge q coulomb are suspended from a                                                                                        the                         repulsive                 between them is
                                                     point by                                                         that                 electrical                         force
insulating  threads each metre long but of
                                           negligible mass. If                                                        equal          to    the weight of either.                            [CBSE D 94]
                                          l
0  is the angle,  each thread makes with the vertical when                                                                                                                                    (Ans. 11.8 cm)
equilibrium          has been        attained,           show        that
                                                                                                                4.       How         far   apart should the two electrons                                be,     if   the
              q =(4 mgl                    sin      0 tan 0)          4n E               (Punjab     95]                 force     each      exerts      on the other is equal to the weight
    Solution.              The given                                                                                     of    the     electron         ? Given that e =1.6 x 101C and
                                                  situation          is   shown in Fig. 1.17.
Each of the spheres                           A and B           is            upon by the
                                                                          acted                                          m,    =  9.1 x     103 kg.                                                 (Haryana 02]
          () its           weight mg,                 (ii)    tension        T in   the string                  5.       A pith-ball  Aof mass 9 x 10   kg carries a charge of
                                                                                                                         5uC. What must be the magnitude and sign of the
         (iii)       the force       of       repulsion F given by
                                                                                                                         charge on a pith-ball B held 2 cm directly above the
                            F=        1
                                                                                                    ..i)
                                                                                                                          pith-ball         A, such that the                      pith-ball          A     remains
                                   4nEn AB?                                                                              stationary             ?
     From (i) and                  (ii),      we have                                                               9.     A     small brass sphere having a                              positive        charge of
                                                                     AC                                                       1.7x 10           Cis made to                touch anothersphere of the
                               1
                                            =mg x                    OC                                                       same radius having                  a   negative charge               of   3.0x    10C.
                                        AB?                                                                                   Find        the       force      between          them          when they are
     But AC=I sin                    0,   OC=lcos 0, AB=2 AC=2lsin 0                                                          separated by              a   distance of        20 cm.         What will be the
                                                    sin 0             I
                                                                                                                              force    between them                   when they        are immersed                   in   an
                                            =mg                 x
                                                                                                                                                                             3?
                               412   sin? 0         cos 0             l
                                                                                                                              oil of dielectric             constant
                 4n    E
                                                                                                                                                                                                                           N)
                                                                                                                                                             (Ans. 1.1x         10     N;        0.367      x    10
                                                    =(4 mg            I     sin    0 tan     0)   4n   Eg'
  I.l6                                                                                                    PHYSICS- X1
 11.        Two
                                                                                    (Ans.    5C          2iC)                                                                                              (FHaryana 20021
                       point charges q,            5x 10 °C           3x10 C HINIS
           are    located            at
                                                             and q,
                                          positions (1 m, 3 m, 2m)
                                                                    and (3 m,
                                                                                                                                                                                                  (Ans. 2.357
                                                                                                                                                                                                                   10G
           5     m,    1   m)    respectively.               Find the forces                                            1.         F=         1
                                                                                             h    and
           using vector               fornm        of   Coulomb's                 law.
                                                                                                                                                                  AT         AT
                                           IAns.
                                                                +2 -k)N,
                                                         hp=-5x 10                     (2i
                                 2cm                                               1   cm
                                                                                                                        3.    For a proton, m                     =    1.67      x 10          kg,
                                                                                                                                         9=+e=                1.6 x    1019 c.
                                                 Fig.   1.18                                                                      Weight of proton                      =    Electrical          repulsive force
           (i)   What           electric         force does                                                                   Or                                          9x9
                                                                         C exert       on    B?                                                                   mg = k.
        (ii)     What           is   the net        electric           force      on B?
                                                  [Ans.(i) 12.0x 10°N,along BA                                                                2kq'9x10 x(1.6 1019                                  x
       ing a charge
                                       metallic spheres                   A and         B, each    carry
                                                                                                                                                          16.36
                                                                                                                                                                       x   10         =  0.014
                                     q, repel   each other with a force F. A
       third                                                                                                                 or                   r= 0.118 m
                      mnetallic           sphere C of the same size, but                                                                                                     =    11.8   cm.
                                                                                                         un
       charged, is successively made to touch
                                              the spheres                                                          4.
                                                                                                                                                               exe
                                                                                                                                         m, g             k.
       A and B, and then removed away.                                            What is     the force
                                                                                                                                                      =
                                                                                                                                                                  2
       of repulsion between                                  and       B?
                                                         A
                                                                                         (Ans. 3F/8)                                                               9x 10 x(1.6                 x 10-19y
14.    Two        point charges
                           +9eand + e are kept at a distance                                                                                                                                               -25.84
                                                                                                                                                                             9.1× 10Sx9.8
       a   from each-other. Where should we place a third
                                                                                                                                          r=       5.08       m.
       charge q on the line joining the two charges so
                                                                                                        that
       it may be in equilibrium ?                                                                                  5.     The pith-ball                   B must have charge opposite to
                                                                                                                                                                                                                   that of
                                                                                                                                  A so   that      the        upward             force    of     attraction      balances
                                                                         3
       suspended                 from the same point by means of                                    silk                                      4,-7.84             x    10         C=7.84          pC.
                                                                      ELECTRIC   CHARGES AND            FIELD                                                                      1.17
 6.   F=
                 1
                                             .103 9x 10
                                                         5 x(0.10)
                                                                     xq'                                                 1    4142
                                                                                                                                     2
            q=       23.9    x10         C
                                                                                                                     9x10 x5x 10 x3x10                                 (2í +2j-k)
 7.   Proceed as            in illustrative           problem on page        1.18.
                                                                                                                                        3                                          3
 8.   Thetwo spheres will                      share the     final   charge equally.                            - 5x10           (2í    +2,-kyN
      Let q be the charge on cach sphere.
                     F=
                                 1
                                             492 =0.025                                           Also,         F    - -  =-5x10(2í+ 2í -kyN.
                                                              N
                                                                                           12.    Here      AB      =2cm =0.02 m, BC - 1Cm =0.01 m
      or
                     9x 10 xqxq
                                                    =0.025                                                                                            B
                                                                                                  A
                            (0.90)                                                                                              FgC                            FRA
2cm cm
                                                                                                                                                                   1
                             0.025 x (0.90)2
                                                         =225 x 10-14
                                     9x       10                                                                                    Fig. 1.20
      or                                 10 6 C.
                     q=1.5           x
                                                                                                  Let q be the charge on each object.
                                                                             )m                                                                           d-x
11.    Here   -í+3f+2k )m,                                   -(3 +5,+ Ê
                                                                                                                                      Fig. 1.21
                 h-i-(3i+ 5,+k)-(i+ 3j +2ê )                                                          For equilibrium            of     charge    q.
-(2í + 2j-k)m
            I-2'+2+(-1                                   =3m
                                                                                                      For equilibrium of charge                   24.
                                         2í+2-k
                                            3
                                                                                                                         k9X24Qx 24                                                           .1)
                                                                                                                                                (d-x)?
                                                                                                     PHYSICS-XI1
                                                     V2+1 d-(V2-
                                                                                    1)d                       static    force   is   the
                                                                                                              charges at rest while the
                                                                                                                                             force
                                                                                                                                        gravitational
                                                                                                                                                       of     attraction          or repulsion
                                                                                                                                                                                         force
                                                                                                                                                                                                  force.
betuween
                                                                                                                                                                                                    isthe force
                                                                                                                                                                                                                  o
                                                                                                                                                                                                              Electva
                                                                                                                                                                                                                      of
          ie, the charge               Qmust be placed                          at a distance         of      1troction        between two                bodies         by     virtue           their mioee
                                                                                                                                                                                           of
          (V2-)d from                 the charge                 q.                                               Similarities               :
  16.    Suppose the three charges are placed as                                        shown in
         Fig.      1.22.
                                                                                                                      1.   Both      forces      obey inverse square law                               ie.,
                                                                                                                                                              1
                                                                                                                                                 Fa
                  A
                                                                                                                  2. Both             forces          are         proportional                  to   product         of
                                                                                                                           masses or          charges.
                                            Fig. 1.22
                                                                                                                  3.    Both are            central       forces        i.e.,   they act along the               line
        equilibrium, the net                         force            on other two charges                        4.    Both         are conservative                    forces     ie.,        the work        done
        should also be zero.                                                                                            against        these forces                    does not depend                  upon      the
             4n    E    (2r)²
                                                      q20                                                                         forces         can operate                 in   vacuum.
Dissimilarities:
1 AC distance r is
OC ke
                                                                                                                                     )
                                                                                                   electrical          forces        are enormously            stronger than the                      gravi
 where m, and                    n,   are the masses of the proton and                             tational      forces.
 electron.
                                                                                                         Examples                       A   plastic       comb         passed through hair
      Hence
                                                                                                   can     easily        lift    a piece of paper upwards The electro
                                        ke?                                                        static      attraction          between the comb and the piece of
                                                                                                   paper overcornes the force of gravity exerted by the
                                  Gm, m,                                                           entire earth            on        the paper.
      But    k   =9x     10 Nmc?, c=1.6x 10                                       C,                     (i) When               we hold          a book    in    our hand, the electric
         m, =1.67 x 10                        kg, m, =9.1 x 10                  kg,
                                                                                                   (frictional)           forces between                 the palm of our hand and
                                                                                                   the book easily overcomethe gravitational                                          force on the
              G=6.67         x   10           Nm² kg2                                              book due to the entire earth.
                             6.67 x          10x1.67 x 1027 9.1x 1031                              length from your friend and instead of being                                                    electri
                                                                            x
enormously         stronger than the                       gravitational          forces.                Principle              of    superposition             of   electrostatic                    forces
                                                                                                   Coulomb's law gives                         force between            two point
     (b)Themagnitudeof the                                                                                                                                                                       charges.
                                                        electric       force exerted by a          The     principle of superposition
proton  on an electron is equal to the magnitude                                         of the                                       enables us                                           to    find the
force exerted by an electron on a proton. The                                           magni-     orce ona point charge due to             point charges. a   groupof
tude of      this force is                                                                         This principle is based on the property that the forces
                                                                                                   with which two charges attract or repel each
                                                                                                                                                 other are
             F= ke' 9x                  1o    x (1.6 x1019,?
                                                                                                   not affected by the presence of other
                                                                                                                                         charges.
                                              (1010,2
                                                                                                         The principle                  of superposition                states        that         when       a
                                                                  [::   r=1        =10       mn]   number               charges        are interacting,         the total force              ona given
                                                                                                                  of
                 -2.3    x   10N                                                                   charge is        the vector          sum      of the forces       exerted         on     it   due      to all
Acceleration of the electron due to the mutual other charges. The force between tuo charges is not afected
                  m          9.1x        10        kg
                                                                                                   413           3e       N          placed        in    vacuum          at          points           whose
                                                                                                                                            individual charges
                                                                                                                                                                       forces         exerted
                                                                                                                                                                                     443
                                                                                                                                                                                                            on
                                                                                                                                                                                                            4N
proton.                                                                                            respectively.
                                                                                        PHYSICS-XII
 I.20
                                                                                                                          force       on      ath charge
     4Y                                                                                                 F=Total
                                                                                                                                                     4TEig b-1
                                                                                                                         ba                                        ba
                                    12
                                                                                                                 N.
                                                                                              where a =1,2,3, ..,
                                                                                                                        for each choice of a. tho
                                                                                                 It may be noticed that
                                                                                                                                           because
                                                                                                                    the value a. This is
                                                                                              summation on b omits
                                                                                                                           over other charges. The
                                                                                              summation must be taken only
                                                                                                                             in a simpler   Way as
                                                                                              aboveexpression can be written
                                                                                              follows :
                                                      93
                                                                                                           =Total
                                                                                                           É             force       on      charge q          due tomany          point
                                                                                                                charges
               Fig. 1.24       Superposition          principle        :Force on
                         charge     q, exerted        by    4,   and q,:                                                                          7-7
     According            to   Coulomb's              law, the force exerted on                                 4Te0     all   pont          |7-7
                                                                                                                         charges
charge       4,   due    to    4, is
                                    1
                                  q92
                     F2         Ae 22T2                                                      Examples based                      on
                                                                                                  Principle      of Superposition of
                                    1            9192                                             Electric      Forces
Formulae Used
-4142
                                                                                                        F=R+                     +2 F         E, cos      0
where fo          =12=a                       unit    vector         pointing from 4, to      Units    Used
                                                                                                       Forces are in newton, charges in                                coulomb     and
                                                                                                                         1                    1
                    1                                                                                   F= q40
                               9142                        + 4,93                  +.....
                                                                                                                                                                   1
                                                                      + 49N                                    -9x 10 x 4x 1o                     x1
                                                                                                  Sum    of the      infinite        geometric           progression
or
                                                                                       ,
                   4Tio
                                                                                                                 a           1           4
                               i-   2
                                                                                               4,
              qat the    vertices           of an equilateral              triangle          of side           What        is   the
                                                                                                         l.
              force on        a charge        Q   (with the              same sign            as q) placed                at    the
              centroid      of the      triangle           ?                                                          [NCERT)
                   Solution.            Suppose the                      given charges are placed as
              shown         in   Fig.       1.25(a).
                                                                                                                                                                                        Fig. 1.26
                                                                                                 ÁO
                                                                                                                                                 Let   Q      be    the      charge required                     to    be kept           at the
                                                     F
                                                                                                                         BO
                   B                                                                                                                        centroid G. Then,
                       42
                                               (a)                                                                 (6)                       F   =Force       at    A due        to    the charge at B
                                                                    Fig. 1.25                                                                           1
                                                                                                                                                                    along BA
                       Let AO= BO =CO =r
                       Force on charge Q due to
                                                                                                                                                                                                                       1       4
                                                                                    q,y                                                        E,=    Force   at   A    due      to    charge at C           =                      along     CA
                                                               1         Q4;
                                                                                 AO
                                                                                                                                             F+=2              cos 30°,along             GA =          W3.-1                              GA
                                                                                                                                                                                                                           2 along
                         Force        on charge Q due to                             4,
                                                               1
                                                                       Q42 BO                                                                    Force at          A due to           charge at G
                                                          Ane,         BO2                                                                                          1                        1                                 1    3Q4
                         Force         on charge                   Q   due to q                                                                                4TEn           AG         4nE,          (l/N3)              4E
                                                                                                                                                 This must be equal and opposite                                      to   (E+      E,).
                                                                                                                                                                                       or
                          By     the principle of superposition, the total force on
nd
                   charge          Q is
                                                                                                                                             Example 22. Consider the                            charges     q,   q and            -g   placed   at
                                                                                                                                             the vertices      of   an      equilateral          triangle,   as shown in Fig. 1.27.
al to                                                                                                                                        What      is   the force       on   each       charge      ?                                [NCERT)
      m,                                       Q9
                                                1[AO +BO+CO]                                        [:        4 =42=43 =9l
=4                                                                                                                                                   Solution.          The        forces         of   attraction              or repulsion
of    1   C                                                                                                                                   between         different pairs of charges are                                        shown        in
[TT 95)                       As      shown in                 Fig. 1.25(b),              the angle between each                              Fig.   1.27.    Each such force has magnitude,
                                                               AO+ BO + CO =0                                                                                                                439
                                 Hence         =0i.e, the                total       force    on charge Q is zero.
                         Example             2Y.Three                  point charges          +q each              are kept      at the
                                                                                                                                        so
                          magnitude and sign of thecharge tobe kept at its centroid
                         that the charges at the vertices remain in equilibrium.
                                                                                                                     (CBSE F         2015)
                                                                                                                                                                                                                           B
                                  Solution.               At       any   vertex,            the     charge               will    be     in
                                                                                              where a             1,2,3,        ,N
                                                                                                 It may be noticed that for each choice ofa he
                                                                                                                                                                        a.
                                                                                              summation must be taken only over other charges The
                                                                                              above expression can be written                                      in   a simpler way              as
follows :
Formulae Used
                                    1
                                          4192
                                  4T
                                                                                                           F-JR++2h                                    H cos 0
       Hence the           total   force on charge        4, is                               Example 19. An infinite number of charges each equal to
                                                                                               4uC are placed alongxaxis at x=1m, x =2 m, x4m,
                       1
                                                                     4;N fN                   X    =8m and        so   on.    Find the            total force       on a charge           of   1   C
                                                                                              placed      at the origin.
                       1
                                                                                                           F=
                                 4,42                + 4,4%
                     4nlig              2 Ir                                                                                                      3
                                                                                                                                        4
       Ingeneral,           force F, on     ah charge 4,       located           at   rdue
                                                                                                           Fe9x 10              x       4 x   10, 3                4.8 x     10 N.
  to   all   other   (N         - 1) charges may be       written           as
                                                                               ELECTRIC                CHARGES AND                FIELD                                                               I21
Example 20.Consider three                             charges         q,,4:4, cach              equal     to
qat     the vertices   of    an     equilateral          triangle             side        What is       the
                                                                         of
                                                                                     l.
forcc    ona chargeQ(with                       the    same sign         as g) placed              at the
centroid      of the   triangle           ?                                                     [NCERT|
      Solution. Supposethe given charges
                                         are placed as
shown     in Fig. 1.25(a).
CO
                                                                                                                                                        Fig. 1.26
                                                                         AO
                                                                                                                   Let Q be the charge required to be kept at the
    B                                                                                           BO              centroid G.Then,
                                                        43
                             (a)                                                          (b)
                                                                                                                    =Force         at   A due     to   the charge         at    B
                                               Fig. 1.25                                                                      1   q               BA
        Let   AO= BO =CO=r                                                                                              4E 2along
        Force   on     charge             Q   due to         4                                                   =      Force at        A   due   to   charge    at   C     =                      along     CA
                                       1 091
                                             AO                                                                                                                                       2
                                               AO?
                                                                                                                                                         GA =V3.
                                    4nEn                                                                                                                                       1
                                                                                                                i+E, =2             cos 30°,   along                                          along   GA
        Force   on charge             Q due to               q
                                      1
                                                            BO                                                      Force at            A   due to charge        atG
                                   4re BO2                                                                                              1                                 Q4                 1     304
                on charge
                                                                                                                                                         4ne (/N3
        Force                                 due
                                          Q            to    43
                                                                                                                                    47E, AG                                                  4rE
                                      1         Q93 -Co
                                                                                                                    This must be equal                  and opposite                to    (E+ E).
        By the principle of superposition, the total                                            force     on              3Q9 =-V3q                    or
                                                                                                                                                                Q
 charge     Qis
                                                                                                                Example 22.Consider the                       charges          q, q   and     -q    placed    at
        As shown         in
                             [A0+ BO +CO)
                                   Fig.       1.25(6),
                                                                         :
                                                             the angle between
                                                                               4 =44=45 =9l
                                                                                                       each
                                                                                                                the vertices
                                                                                                                What
                                                                                                                between
                                                                                                                Fig.
                                                                                                                         is
                                                                                                                       1.27.
                                                                                                                                   of   an
                                                                                                                                  different
                                                                                                                                             equilateral
charge
                                                                                                                                                               of
                                                                                                                                            pairs of charges are
                                                                                                                                  Each such force has magnitude,
                                                                                                                                                                      ?
                                                                                                                                                                            as shown in Fig.
                                                                                                                                                                      attraction              or repulsion
                                                                                                                                                                                                   shown
                                                                                                                                                                                                           1.27.
[NCERT)
in
                                   A0+ BO + CO -0                                                                                                           439
          Hence         =0ie.,the              total    force on charge                    Qis zero.
                                                                                                           so
magnitude and           sign        of the charge to be kept at                        its   centroid
|CBSE F 2015]
                                                                                                                                                                                         B
        Solution.        At any                vertex,            the charge               will      be    in
                      V2F212)                                   BC        -FBC
                                                                                                                  tan   f
                                                                                                                                     L, sin     120
                                                                                                                                                                     180+
                                                                                                                                                                          180     sin
                                                                                                                                                                                  180 cos 120
                                                                                                                                                                                             120
                                                                                                                                                  120
                                                                                                                              h+Lcos
where BC             is    a unitvector along BC.
                                                                                                                                     180    3/2                  V3
     Similarly.total                force      on       charge q,             is
                                                                                                                              180+ 180(-)
                                -FAC
                                                                                                                        B= 60
where AC             is    a unit vector along                       AC.                                                                               F     is parallel              to BC.
                                                                                                           ie.,    the resultant force
     Total force               on charge           q,   is
                                                                                                          Example 24. Four          equal point charges each 16 uC are
                                                                                                                                          a square of side 0.2 m.
              H=F+ F+2FxF                                     cos   60°        n   =V3 Fn                 placed    on the four corners
                                                                                                                                                                  Calculate
                                                                                                                                                       of
                                                                                                          the   force onanyone of the charges.
where n             is a       unit vector along the direction                            bisecting
                                                                                                                Solution. As shown in Fig,                                    1.29,   suppose the                  four
= 180 N, along BA
                                                                                                                            Bg                  0.2   m
                                                    A120°
                                        +5 uC
                                                   60                                                                                             Fig. 1.29
Force exerted on 4, by q, is
                                                                                                                                           9x 10      x 16 x             10      x 16 x      10
                           B
                      +10 pC                   5cm                        10   C                                                                                 (0.2)
                                           Fig. 1.28
                                                                                                                                      = 57.6 N,along AD                         produced
                                                                                                             Force exerted              on 4, by            4,   IS
                                                                              A with a        force
     The charge at Cattracts the charge                                  at
                                                                                                                                           9x   10 x         l6      x   10x          l6 x       10
                                                                                                                                                                18 x          10
                                           F=   + F=                       28.8       +   81.5
equilateral triangle of side 20 cm as shown in Fig. 1.30. the x-axis at points x =10 cm, 20 cm, 30 cm, ...,
              placed    at the midpoint            (M)                of side         BC so that         the     charge at     A                   the   second          8x        10C,third                   27 x   10       C, and so         on.
6 uC    are
                                                                                                                                                                                                                                  10* C. Find
               remains      in equilibrium                ?                                                      [CBSE D 05]                       The   tenth       particle             has       a    charge 1000 x
Calculate
                                                                                                                                                   the magrnitude of the electric                               force      acting       on   a   1C
                                   +2uC                                                                                                                                                                               (Ans. 4.95 x 10° N)
                                      A                                                               A+2uC                                        charge    placed             at the origin.
 the four
                                                                                                                                                   Charges q             =           mC, q, = 0.2 mCand q, =- 0.5 mC
                                                                                                                                                                                                                                      ,
                                                                                                                                              2.                              1.5
re   ABCD.                                                                                  F
                                                                                                                                                                                                              B and C respectively, as
                                                                                                                                                   are placed            at    the points A,
                                                                                                                                                    shown       in       Fig.        1.32.          If    r   =12 m and = 0.6 m,
                                                                                                                                                    calculate        the magnitude                       of resultant          force    on   4,
                                                      1            4192
                                           E=                                                                                                  AA
                                                                                                                                                                                                                           B
91 92
                                                 9x       10           x2 x 106x3                      x 10-6
                                                                                                                                                                                              Fig. 1.32
                                                                                (0.20)
                                                =1.35         N, along AB
                                                                                                                                               3. Two equal                positive                charges,    each of uC interact2
                       Force exerted on charge                                  +   2uC by        charge         at C,                              with a        third         positive                 charge of      3
                                                                                                                                                                                                                     uC situated as
  10C                                            9x           10       x    2 x 10 x3x                    10-6                                      shown         in          Fig.        1.33.          Find       the magnitude                and
                                                                                                                                                     direction       of the force experienced by the charge of
                                                                                    (0.20)                                                           3uC.            (Ans. 3.456 x                       10    N, along OC produced)
 106                                            =1.35      N, along AC
                       Resultant force            of      E and E                                                                                                                  2      C
ced
                                            F=F+ E,+2 F,E                                             cos 60°
                                                                                                                                                                  3m
                                                =1.35+                          1.35       +2x         1.35    x 1.35 x 0.5
                                                                                                                                                                                                                      3 uC
10-6
                                                =V300                  =   10/3 cm
ed                                              =0.1x             3m                                                                                                                           Fig. 1.33
       24                                                                                      PHYSICS-XI|
HINTS
                                                                                                                   Similarly,      force      exerted by charge q, on                                  qc.
  1.   By the principle of superposition,the                                     total             on
                                                                                           force                              F,         2.16x 10                  N, along            BC                                         There is a d
       the Cchargeplaced at the origin is
             1                                                                                                                                                                                produced
                                                                                                                  Clearly,        F,F             (in        magnitude)                                                        the above equa
                                                                                                                  The components                   of    F and Ep along                                                        charge distrib
                                                                                                                                                                                                     Y-axis       will
                                                                                                                  cancel     out and               t   added along X-axis.                                                     change the         elec
                                                                                                                                                                                                                                    Units
                                                                                                                                                                                                                                            test    ch
                                                                                                                                                                                                                                                  and
                  1      9293_9x10                  x0.2 x            10      x0.5     x   103                                                                                                                                 electric    field       is
                                                                                                                                                                                                                   it.
                                                                                                                                                                                                                               follows :
        F=JE +                      -3.125 x10° N.                                                             Consider a charged body                         carrying            a positive               charge q
                                                                                                        placed at point 0. It                      is    assumed  that the charge g                                                          (E] =
 3.    Here 94=98                 =2uC=2 x 10° C,                                                       produces an electrical                         environment in the surroun
                   9c=3uC=3x106c                                                                        ding space, called                electric field.
                                                                                                                                                                                                                               ditferent     point
        3m                                                                                              produces the             field       E.
                                               9c                                                                                                                                                                              In general,         E
                       4m
                                                                                         -X                 29. Define electric               field          at   a point.         Give          its    units     and
                                                                                                                                                                                                                               vectors.      Each
                                                                                                        dimensions.
                                                  52
                       =    2.16     x 10      N,           along          AC     produced
                                                                                                                                                                                                                                       Electrosta
                                                                                                      ELECTRIC                CHARGESAND                FIELD
                                                                                                                                                                                                                                   1.25
                     not change the value of E. lt                            is   better to define glectric                         charges and    independent of the test charge that
                                                                                                                                                           is                                                                              we
                     field as follows :                                                                                                                               field.
                                                                                                                                     place ata point to determine the
                                                                         E     can          be    determined                    as   drop    carrying       10electrons when                         located      between         the plates
                        The dimensions                         for
d around       it.
                     follows :                                                                                                       of a capacitor        which are 5 nm apart.                          The       mas       of oil drop   is
                                É() at every              point r                                                                     Example 27. A                  positively              charged oil drop is prevented
 charge                                                                                                                                                    under                            by applying a vertical electric
he     electriC
                9o
                                                                                                                                              of 100     V.          If
                                                                                                                                                                          gravity
                                                                                                                                                                                   E=100 Vm-l
                             Electrostatic             force    = Charge                x   Electric      field.
     I26                                                                                        PHYSICS-XI1
                                                                                                                                         -
                                                                                                                                                                                                  Sin
due to the electrostatic force acting on the sphere, neglecting sin 0 sin 27 Time of fall of the prot
       For small 0,           sin   0-0=                                                                2x      10     NC-.              Calculate          the time of travel.                     The mass and
                                                                                                        charge         of        electron            are     9x 10               kg        and       1.6x 10 C                                    -1.25           10     s.
                                                                                                        respectively.                                                                                     [CBSE D 91]                  Thus the heavier partie
                                                                                                               Solution. Force exerted on                                            the electron by the                        through the     same distan
                                     A                                                                  electric        field,
                                                                                                                                                                                                                                situation          of   free fall unde
                                                                                                                                         F=     eE                                                                              fall     is    independent of                  the
                                                                                                                                                                                                                                                                                   pl
                                                                                                               Now        u =0,s =6.0                  cm      =0.06           mn,    a =0.35 x               10°ms             20 mm. The upper                   plate has
                                  Fig. 1.38               qE
                                                                                                                                 0.06         =0   +x           0.35 x         10l6 x
                                                                                                                                                                                                                                        Solution.             Here         V=
                                                                                                                                                      0.06     x   2                                                             e
                   ma     =                                                                             Or                                t=                                   =0.585        x    10s.                               =L8x 10                   Ckg
                                                                                                                                               V0.35        x 106
                                                                                                                                                                                                                                        Lpward           force         on      the
or                d'xqEy=-ox
                      ml
                                                                                                        Example 31. An electron                                falls   through a             distance           of 1.5    cm     field     is
                                                 1                                                                                                                                  eE
      As         u   =0,       s=ut+-at=                                                                                                         1ma   = eE        or         a=          and t=
                                        2
      ..   Time of             fall    of the electron                     is
                                                                                                                                                   y=
                                                                                                                                                                         2 m             v
                          |2s          2sm,                      2x 1.5x             10        x9.1    x      10-31
                                                                                                                                                                         of
                                                                                                                           deviation      of the path       of the   electron           as it comes out of the field.
                          2s                                     |2x 1.5x            10-×1.67                 10-27
                                       (2sy                                                               x
                     =1.25x          107 s.
   Thus the heavier particle takes a greatertime to fall
through  the same distance. This is in contrast to the
situation of 'free fall under gravity' where the time of
fall    is   independent                    of       the mass              of    the body. Here the
                                                                                                                                                                     Fig. 1.41
acceleration               due        to   gravity               'g',   being         negligibly               small,
has been ignored.                                                                                                                 Solution.            Acceleration of the                         electron               in the
Example 32.An                         electron             is    liberated       from the lower of the                     upward direction,
two large            parallel         metal          plates         separated             by    a   distance          of                            a= eE
20 mm.           The upper            plate      has            apotential        of   +2400 V relative
 to the      lower        plate.      How        long does               the    electron        take      to reach
                                                                                                                                  Time taken           to   cross the field,                  t=
       upper          plate     ? Take                of    electrons           1.8   x   1o" Ckg-1.
the
                                                                                                                                  Upward         component of                            electron             velocity            on
       Solution.                Here V               = 2400 V,                   d=20 mm             =0.02            m,   emerging           from field region,
                                                                                                                                                                     eEl
       -1.8 x        10" C           kg
                     md                    0.02
                                                                                                                            Example 35. A                charged particle, of charge                   2uC and mass
           Using,      s  =at,
                           2                     we    get                                                                  10 milligram,              moving with            a velocity         of 1000 m/s entres a
                                                                                                                                                                                                             NC-
                                                                                                                                                                                                   10                     directed
                     2s         2d                   2 x 0.02                                    10s.
                                                                                                                            uniform        electric         field       of     strength
                                                                               S=1.4        x
                                                                                                                            perpendicular          to   its   direction            of    motion. Find               the    velocity
                                                                                                      a                                                                                                       normal         to   the
 Example 33.                     A   stream of electrons                    moving          with              velocity            Solution.        The velocity               of the         particle,
                                                                                                                                                                                      19                  10
                                                                                                                                                      cE             L6x         10          x3.34
                                                                                                                                               F
    The          velocity         of the       particle,             along the         direction                 of
                                                                                                                             3.    ()a                                                              27
                                                                                                                                              m                                  1.67x       1
                                                                                                        1
                                                      10× 106                                                                                                                                                                            the
                                                                                                                                   s0}at'
                                                                                                                                  ()
    The net velocity after                            10                                                                                                                                                                                 the
                                                           s,
                                                                                                                                                                                |2002 -3.54                       107
         p-vI000)+(2000)                                                            -1000 V5         ms           !                                                         V3.2        x 10T                                            sym
                                                                                                                                                                                                                                         Sam
    Displacement, along the -axis,                                        after      10 s,                                        (i)The field must act                           vertically            downwards so              that
                                                                                m
                                                                                                                                                                          thrown against the                     field,   so
    Displacement alongy-axis                                   (in    the direction             of field)                  4.      As the particle is
after 10 s,                                                                                                                               F      qE                             40x10x10                         4x10 ms 2
                                                                                                                                                                                 100x 106                                                po
  yet4y? =
                    1
                                  (0)    +1y212x10x10
                                              2   m             ,10x10                            x(10)
                                                                                                                                      7=+
                                                                                                                                                11
                                                                                                                                                      2as
                                                                                                                                                             m
                                                                                                                                                                                                                                         Con
    -10000               m
                                                                                                                                    0 -(200)' +2(-4×10')xs
    Net displacement,                                                                                                                         200 ×200                                                                                   hav
                                                                                                                                                                 m              0.5
                                                                                                                                                                                      m.
                                                                                                                                               8x104                                                                                     orig
         r=yr'y? - (10000° +(10000) =10000V/2                                                               m.
                                                                                                                                                             3.2 x 1019               x16x10 =8x102ms2
                                                                                                                             5.    a= F_qE
                                                                                                                                         11                               6.4x1027
        Problems For Practice
                                                                                                                                               -i=2as
    1.       Calculate the electric field strength                                  required       to just                                u'-0=2 x8 x102 x2x102
            supporta water drop                        of       mass 10° kg and having a
                                                                                                                                                     v=4/2                x10         ms1.
            charge 1.6 x 10-19 c.                                                      [CBSE OD              99]
                                                                                            106 NC-l)
                                                                     (Ans.6.125        x
                                                                                                                      1.19 ELECTRIC                    FIELD              DUE TO A POINT CHARGE
    2.      In    Millikan's experiment, an oil drop of radius
                                                                                                                             31. Obtain               an         expression for                         the     electric       field
             10  cm remains suspended between the plates                                                                                                                                                                    What
                                                           of 5e                                                                       at a point at a distance r from a charge q.
            which are 1 cm apart. If the drop has charge
                                                                                                                      intensity
                                                           x 10°NC-,
            uniform electric field of magnitude 3.34
                                                the             (ii) the                                                                                                                                                                  poir
            Determine (i) the acceleration  of       electron
                                                                                                                                    Source
                                                                                                                                                                                            Test
            time taken    by  the   proton   to  fall    through     the
                                                                                                                                     charge
                                                                                                                                                                                                                                           Cou
                                                        of the electric                                                                                                                    charge
            distance of 2 cm, and (iii)the direction                                                                                                                                                                                     chai
field. Mass of a proton is 1.67 x 10kg. Fig. 1.42 Electric field of a point charge.
                                                                                                                                                                                                                                         are
                                                                           ELECTRIC             CHARGESAND              FIELD                                                                  .29
   Clearly, Exl/        This means that at
the spherical surface
                                             all points on                                                     Hence,    the electric                field       at   point   Pdue to the      systern
                        drawn around the point charge,                                                of   N   charges         is
the magnitudeof E is
                         same and does not depend on
the direction of r. Such a
                             field is called
                                                spherica lly
symmetricor radial field, ie., a field which looks                                                                           1
                                                        the
same in all directions when seen
                                  from the point charge.
                                                                         E;
                                                                E,+
           91
point P whose position vector is r. According to In terms of position vectors, we can write
charge 4,         is
                                   1                                                                                                 4neo
                                                                                                                                                    i=1|7-77-l
                                                                                                                                          1
                                4Te0
                                   1
                                                                                                            2.     By the         principle                 of    superposition,       electric       field
                                                                                                                                                                        of
istribuied uniformly in a sphere of rodius 10               hat         & cm
                                                       m         placed       apart in air. Determine the posilion of lhe pornt
twll be the electric field at the
                                  surface  of the gold          at ehich the resultant    field is zero.
                                                       atom
 For  gold.Z-79
                                                                                                              Solution. Let             P be the         point at                  distarice        x    cm   from
   Solution. The charge may be
                                    assumed to becon                                                          where the net             field     is   zero.
                                                                                                         A,
Centrated at the centre of the
                               sphereof radius 10                                            m
                             m. q-Ze-79                 x   16x 10C                                           4,16uC                                                                                     -9p
               E
                                 9x10 x                 79 x 1.6x        1019
                                                      (10        0
distance d apart.
                         respectively
                             The
                                       are placed ata
                                       clectric field       at the middle point
                                                                                                              At point         P,                        E    +     E =0
                                                                                                 O
betueen the charges is 6.4x
                                NC          10                   Find the         distance     'd
                                                                                                                kx     16x    10            kr(-9)            10
                                                                                                                                                                               =0
betueen the point charges A and
                                 B.                                   (JEE   Main       June   22]               (xx     102}              [(8-x) 1o
     Solution.
                                                                                                                                                                    16                   9
       -810            C
                                                                        -8x        10    c                                                                                            6-x
                                                                             -B                                                                                                         3
                                                              d/2                                       or
                                                                                                                                                                               32
                                                            kq        8k
                                                                       d
                                                                                                        or
                                                                                                                                                                              I=m,
                                                                                                                                                                                7                   32   cm
                                                                                                                             32
                                                                                                                         =
                                                                                                                             7 cm, both
                                                                                                                 At x                                  E,    and E, will be                      in      the    same
                       d2škg_8x9x 10                         x8x10-6
                                                                              =9                        direction,therefore,                     net    electric              field    cannot be              zero.
                   1   1x1o6                1          4x 10-6
                                                                                                                     A
                                                                                                                                                 EE           0.1   m
                                          4n         (0.30-x?
and E, will be in the same direction, therefore net                                                                               kq,           9x10         x0.2
electric field cannot be zero. Net electric field  will be                                                               E                                                   -72       x     10l NC!,
zero at x=+0.10m ie,at 10cm to the right of +1uC                                                                                                  (0.05)
    Electric      field        atO due to               q                                                                The clectric          field       vector E, at                A due           to   the negative
                  ky,              9x10        x0.4
                                                                                                                   charge 4, points towards                                    the      right           and       it       has     a
                                     (0.05)                                                                         magnitude,
-7.2 x 10" NC,acting along BO. Magnitude of the total electric field at A L
    (b) Electric                                                                                                              E, =    E +        E,
                          field       at P      due to           q,                                                                                                                =7.2              10       NC1
                                                                                                                                    3.6x         10 +3.6x                   1o
                               9x10             x 0.2
                  kq,
                                      (0.15)
                                                            ,   acting along                 AP
                                                                                                                         E,   is directed            towards                theright.
    Electric        field      at P       due     to        b                                                         The electric              field      vector E,at                      B   due     to    the positive
                                                                                                      -p                                      (0.05)?
        A                                                                                    E,
                                          m                                                                              The electric           field      vector E, at                B due to               the negative
                                   -0.1                                           0.05       m
                                                                                                                   charge          q5    points           towards                the    right            and       it      has      a
                                                                                                                    E,   =NE+ E +2                                   E,cos       0
                                                                                                                                                               E,
                                                                                                                         =(9x           10    )'+ (9           x 10)+ 2x9x10° x9x 10' cos 120°
                                                                                                                         =9x 10°1+1+2(-1/2) NC-=9x10° NC-1
                      +10°c
             B                                                                               --10c                        Since         E,    and         E, are equal                 in       magnitude,                 so    their
                     0.05      m    -H-        0.05    m               -   0,05    m         -                      resultant                E, acts       along              the bisector of the angle
             k
                                                                                                                    between             E, and E,,                  i.e,   towards           right.
                                               Fig. 1.49
positive charge 4, points towards the right and it has a respectively. Find the resultant electric field at B.
                                                       NC
                                                                                                                                               E      =k=k=2k,along                                               AB
                                   =3.6x 10
I32                                                                                                        PHYSICS      1
                                                                                                                  Exarple 43. For
                                                                                                                                                         charges44-4-4 are                                       placed
                      A               Sm                                                                                 pectielyaf        the   four corners A, B, C and                         D   of   a square      of
         +   50   C                                                                                               side    'a' Calculate          the electric field at the centre                     of the square
Panjab
5m Tig 152
                                                                                                                          A
             Fig. 1.50                                                                                                                                                                                                        Their magnitudes are
                                                                                                                                                                                                                                                        1       6a
    Electric ield at B                   due to         -50          C    charge            at   Cis
                                         50
                                               -2 k,        along BC
                                                                                                                          De
                                                                                                                                                                          D
     Electric field              at   B due to + 50 C charge                                at   D    is                                                                                     (b
                                                                                                                                          (a)
                                                  50
     Component of
                           E,    =k.
                                         s
                                      E,along x-axis
                                                    +5
                                                                     =k,
                                                                      =2k
                                                                                along DB                                                                   Fig. 1.51
                                                                                                                                                       E
                                                                                                                                                                                      2       :+-)                                                    =E52 =
                                                                                                                                                                                                                               (fhe
      .                                                                                                                                                                              same
                                                                      1              k                                   Because          E, and              act in          the                 direction,             so                                  5eld
                                 =E          cos 45°        =   k.
                                                                                                                  their resul tant is
                                                                                                                                                                                                                                            resultant
tions,    therefore,             total       electric       field        at    Balong            y-axis                                                                                                                        2.   Determine the electr
                                                                                                                  directed parallel              to AD or BC, as shown in Fig.                                 1.51{b)
                                =2k      -                                                                                                                                                                                           muceus at a distance
                                                                                                                                        cos B    =                         ..B=45°
                                                                                                                                                     E        V2
         Resultant              electric        field   at      B        will   be                                                                                                                                             3.    Two    point charges
                                                                                                                  ie,    the resultant          field is inclined      at      an angle of45° with AC.                                            of 6a.    Find
                                                                                                                                                                                                                                      distance
              E-E+                                                                                                Example 44.Two point charges +6q and -8qare placed at                                                              bwo chargeswhere
                                                                                                                  the vertices 'B and 'C' of anequilateral triangle ABCof sade                                                                         (Ans.         At
a' as shown in Fig. 1.52(a). Obtain the expression for ()the 4 Two point charges
                                                                                                                  magnitude and (i) the                  direction        of    the resultant              electric field            -2x10Crespec
                  =3k =3x9x 10                        NC=2.7                        x   1010     NC 1             at the vertex A due to                 these     tuo        charges.             [CBSE OD           14C)           Calculate the electr
                                                                                   20
                                                                                                                   expression                for      ()     the      magnitude                         ard      (ii)    the
                                                                                                                                                              A
+6qe                                             8q   + 6qe
                  EgA    = 4TE         1    6q
                                                   =6E, where E =
                                                                                        1
                                                                                                  9
                                            a
                                                                                                                                                           Fig. 1.53
                                   1        8q                                                                                                                                                 of electric         field   at
                                                      -8E                                                    6.       Find the magnitude and                          direction
  tanß      =
                  EAc t EgA                COS120°
                                                                SE+6E
                                                                                                  5                                          Ans.      E=
                                                                                                                                                              4E,
                                                                                                                                                                  1
                                                                                                                                                                      -,   a
                                                                                                                                                                                            along        BP produced
                                                                 from     it.
                                                                                                                                                      and r=1=10                             10 m.
       nucleus at a distance of                                                                                                     q=+
                                                           1
                                                                                                                                                2e
                                                                                                                 2.     Here
                                                                 (Ans. 2.88 x               10!   NC-)
                                                                                                                  3.    Suppose the electric                       field           is   zero at distance x from
                              charges + q and + 4q are separated
                                                                 by                                     a
  3.   Two         point                                                                                                 the charge            + 4. Then
                                      on the line joining the
       distance of 6a. Find the point
       two charges where the electric field is
                                                  zero.
                                           from charge + q)
                                                                                                                                                                      4n       E            (6a -       )
                   (Ans. Ata distance 2a
                                              2x 10°C and                                                                                      (6a-x)          =4r                  or        6a   -x=2r
        Two          charges 4, and 4, of
                   point
                                                                                                                            Or
                                                               are placed 0.4 m apart.
  4.
        -2    x   10C          respectively
                                                                 at the centre of the line                                         Electric          field    is    zero at distance 2a from the
                                                      field
       Calculate the               electríc
                                                                              [CBSE F 94C]
       joining the two charges.                                                                                             charge       +4
                 (Ans.9000                            NC, towards the -vecharge)                                  4.        Proceed as          in     Exercise         18 on                page        1.80.
 I34                                                                            PHYSICS-XI|
                                                                                                                                                        distribution.
                                                                                      exerted by all such
                                                                                      small charges and is
                                                    D                                 given by
Or 1
                                                                                                                                            dq
 1.21        CONTINUOUS                      CHARGE DISTRIBUTION                                                                    p=
                                                                                                                                           dy
       33.   What is
               a continuous charge
                                    distribution ? How                                      The SI unit for p
 can    we                                                                                                                    is   coulombper cubic                metre   (Cm).
                 the force on a point
             calculate
                                      charge q due to a                                     For example,
continuous charge distribution ?                                                                                              if    a
                                                                                      charge q is distributed
     Continuous charge distribution.     In practice,                                 over the entire volume
                                                         we
deal with charges much greaterin
                                     magnitude than the                               of a sphere of radius R,
charge on an electron, so we
                               can ignore the quantum                                 then its volume charge
nature of charges and imagine
                               that the charge is spread
in a region in a                                                                      density is
                   continuous manner. Such a
                                                    charge
distribution is known as a continuous
                                       charge distribution.
                                                                                                   p= 4 9Cm3                                                        dV   +
   Calculation of the force ona
                                charge                       due to a conti                                   TR3
nuous charge distribution.    As                                                                          3
                                                        shown in Fig. 1.56,
consider a point charge q, lying
                                  near a region of contin                                  The
uous charge distribution. This                                                                          charge   Con
                                continuous charge distri-
bution can be imagined to                                                             tained       in   small volume
                            consist of a large number of                              dV    is
                                                                                                                                                         dq   =p dV
 small charges dq.
                    According to Coulomb's law,
force on point charge 4,                               the
                          due to small charge dq is                                               dq    =p dV                                     ig.1.57     Volume charge
                                                                                                                                                       distribution
                                                                          ELECTRIC     CHARGES AND               FIELD                                                                  1.35
          Total electrostatic           force exerted on charge g,                   dueto      density at any                point          on   this     line      as the   charge per unit
      the entie volume V               isgiven by                                               length    of   the line af that point,                     ie,
                                                                                                                                    dq
                                                                                                                                    dl
                                  0
         (b) Surface         charge                                                                                                          dL
                                             distribution.           It    is    a   charge                                                                          dq   = dL
      distribution      spread over         a   two-dimensional            surface    S    in
space, as shown in Fig. 1.58.                            We    define the surface
charge density at any point on this                        surface as the charge per
 unit areaat that point, i.e.,
                                  dq
                                                                                                                      Fig. 1.59           Line       charge     distribution.
                                  ds
         The SI unit        for   is Cm                                                            For exanmple, if a charge q is uniformly distributed
                                                                                                over a ring of radius R, then its linear charge density is
                                                                                                                           2=                  Cm-1
                                                                                                                                2TR
                                                                                                      The charge            contained                in   small length dLis
                                                              +
                                                                                                                       dq     =dL
                                                                                                      Total electrostatic                    force exerted on charge q, due to
                                                                                                the   entire     length Lis given                         by
                                                                                                                                                           dL
                                  dq   =o dS
                                                                                                      Electric      field      due           to   the line charge             distribution    at
                      Fig. 1.58    Surface      charge     distribution.
                                                                                                the   location        of charge 4, is
                        dq =odS                                                                                        1
                                                                                                or    Econt
    Total electrostatic force exerted on charge                                 q%   due   to
                                                                                                                                    V                                              L
the entire surface S is given by
                                                                                                   General charge distribution.  A general charge distri
                                                                                                bution consists of continuous as well as discrete charges.
                                                                                                Hence       total       electric              field       due        to    a general charge
        Electric field      due to the          surface charge            distribution          distribution          at the location                     of   charge       q,is   given by
       the location of charge 4,                is
                                                                                                                              =E
 at
                                                                                                                E     total              discrete               cont
                                            1                                                                                            1
                                                                                                                                                     N
                                                      dSi.                                                          total
vector direted from each point of the volume, surtace                                                 Find    hoe the       surface    density            of   electrification        will   changeif
or line charge               distribution         towards the location of the                         Ho charge is lost.                                                      EE Main June           221
   1        Volune charge density,                       p                                                   Let   Rbe     the radius of the bigger drop formed. Then
                                                                   dV
                                                             ,
                                                                   dS
                                                                                                       Or
                                                                                                                          R'(002yx4'
   3      Linear           change       density,        A
                                                                   dL                                                       R0.02x4 =0.08 m
   4 ForeeNerted on a charge                                        due     to a   continuous                 Charge     on small drop 5uC-5x                                10C
         charge distribution
                                                                                                             Surface charge density of small drop,
                                                                                                                                                          5x10
                                                                                                                                                                        Cm2
                                                                                                                                       4
                                                                                                                                                      4n(0.02)
    5.   Electric field            due to a continuouscharge distribution,
                                                                                                             Surface charge density                       of   bigger drop,
                                                                                                                                      5x 10x 64
                                                                                                                                                                 Cm 2
   Units Used                                                                                                                              4n(0.08)²
                                                                                                                                                     -6
          pis        in    Cmin Cm,à                           in   Cm        and Ein      NC1,                                       5x 10                        4n
                                                                                                                                                                     (0.08)-1:4
                                                                                                                                      4n(0.02)                  5x   10x64 4
                                                                                                                                                               ,
                                                                                                             Solution. Electric field   of a line charge from
        Solution.              o                                                                      Coulomb's law. Consider an infinite line of charge with
                                                                                                      uniform line charge density         as shown in Fig. 1.60.
        n    first    case                    =9
                                            0.7
                                                    4n                                        *)
                                                                                                      We  wish to calculate its electric field at
                                                                                                                                                  any point Pat a
                                                                                                      distance y from it. The charge on small element dx of
                                                                                                      the line charge will be
        In    second        case
                                                                                                                                                     = hdx
                              0.7 +0.14          -9+0.44                                                                                     d
4n
                                                   9t0.44                                                                              dEy
                                          0.84                                               ..(0)
                                                        4nr
        Dividing (i) by (), we get,
                                                                                                                                      d    E,
                       0.84         +0.44                or
                          07                                        514044
             Initial        charge, q-2.2 C.
                                    y07x4x22
                                                                                                                   Fig. 1.60    A section            of   an   infinite line of       charge.
                                                                                          ELECTRIC          CHARGES AND            FIELD                                                                                137
            The     electric        field         at the point                 P due      to    the charge
      element dq will be
                             dE     -         1
                                                       dg            1
                                                                                    hdx                                            11                                                    dE.sin   9
                                                                                                                                                                                                         dE cos 9
           The      field    dE has two                                                                                                                                                                                 dE cos
                                        components:
                            dE, =-dE sin 9 and    dE, = dE cos 0
                                                                                                                                                                                     dE sin       9
           The      negative sign in x-component
                                                                                        indicates that
      d   E, acts in        the negative                x-direction.                Every charge           ele
   ment on the right has a
   out.
                           corresponding charge element
   on the left. The x-components of
   elements will be equal and
                                        two such charge
                               opposite and hence
       The resultant field E gets contribuions                                                  only
                                                                                                     cancel
                                                                                                       froms
                                                                                                                       .
                                                                                                                  element
                                                                                                                           The magnitude
                                                                                                                                dl at        the     field
                                                                                                                                                                 Fig. 1.61
                                                                                                                                                               of the
                                                                                                                                                                point
                                                                                                                                                                              field
                                                                                                                                                                               P is
                                                                                                                                                                                          d E produced                  by the
   y-componentsand is given by
                                                                                                                                                           dq             k         dl
                                 x=+o
                                                                                                                                                                       2Tua2
                             E= E, =[aE,-                             fcos9dE
                                                                                                                      As shown                     in    Fig.        1.61,          the        field         dE has          two
                                          X=00                                                                    components :
                                                                          1             dx
                                 =2                     cos    0.                                                     1.   the    axial       component              dE cos 0,                and
                                          x=0                                                                         2. the perpendicular                     component                 dE sin         0.
                               0 =/2
                                                                              0 de                                           E=              dEcos 0
                                              cos      0 y          sec
                                                                                                                                        0
                                                                                                                                                                 [Only               the axial components
                       2TE0
                                  =0                     y' (1+ tan'e)                                                                                                               contribute towards E]
                                                                                                                                                                                              2ra
                                    0=n/2                                                                                                                 dl     x                        1
                            2                                                                                                                                                 kqx
 plane perpendicular to the x-axis, as shown in Fig                                                    1.61.          This   is   the        same as            the field            due to           a point charge,
 Consider a small element dl of the ring.                                                                        indicating that for far                   off axial            points,           the charged ring
                        dq   =2T      9           dl                                                             charged uniformly and                     the   charge per unit length                           is   à Find
                                          a                                                                      the electric     field       at   its   centre.              [CBSE PMT               2000,    AIEEE       2010]
                                                                                                  PHYSICS-X1
138
                                                      cach                                                 HINTS
                        two symmetric clements
    Solution. Consider                                                                                                                  4
                                               of the  two                                                       1    Use o
of length dlat A and
                       B The electric   fields
                                                                                                                                       4nr
                              PO    get  cancelled   while
                          to
elements perpendicular                                                                                                      q4nr              a
                                                                                                                       Use
            PO      added.                                                                                       2.
those along
   Electric
                get
                 at field           O
                     due to an element of length dl
                                                         is
                                                                                                                 3.   Surface area             of   cube        6r16                      0.01-0%, m2
                                                                                                                      4                      4
                      dE=                 1    d4cos 0                               |Along        Poj           4
                                                                                                                      3
                                                                                                                                                                                23   2      22/2:2
                                          1    2dl
                                                      COs 0
                                                                                                                                       4n2           24           22             22
                                          1    (ad0)cos 0                                        =ad9]
                                                                                           [dl
                                                                                                           1.22 ELECTRIC DIPOLE
                                   4Ten           a                                                                                                            ? Define dipole moment
                                                                                                                                                           dipole
                                                                                                                     35.What isan             electric
                                                                                                                                                                           of
                                                                                                           electric dipole.
                                                                                                                                      is either   charge times the separation
                                                                                                           vector whose magnitude
                                                                                                                                                                         along
                                                                                                                                        charges and the direction is
                                                                 dE                                        between the two opposite
                                                                                                                                                     the positive                                charge.
                                                                                                           the dipole      axis from the negative to
                                                                                                                                                     separated by dis
                                                                                                           consisting  of charges + q and -q and
                                                                                                                                                  is called dipole axis.
                                                Fig. 1.62                                                  tance2 a. The line joining the charges
                                                                                                                                                           2a                                    +4
    Total      electric           field        at the centre            O    is
                                                                                                                                                           p
                     n/2                  z|2        1      hcos      9d0
           E=            (dE=2                                                                                                                        Fig. 1.63
-n/2
or p =qx2 a
Droblems for Practice Thus the dipole moment p is avector quantity. Its
    1       Auniformly                charged sphere                   carries     a total       charge    direction         is   along the dipole axis from                          -q   to    + q and its
          of   2r x      1012C. Its radius                    is     5 cm and         is   placed    in    magnitude              is
                                                                                                                                                          4n6,(r-a'                    (r+a?|
   1.23 DIPOLE FIELD                                                                                                                                                        4ar
charge of an electric dipole is zero. But the electric field (towards right)
+4 P Ex
20
(towards left)
                                                   o   (r+
                                                                                                                                      A                                                           B
                                                                                                                                equatorial       line.
                                at       point           P due        to-charge                is                 along the
   Electric     field
                                                                                                                                                                 an electric
                                                                                                                         Derive an expression for the torque on
                                                                                                                       40.
   Thus the magnitudes                              of       E,and                E,, are equal ie,                                                    field. Hence  define
                                                                                                                  dipole placed in a uniform
                                                                                                                                              electric
                                                         1
                                                                                                                  dipole    moment.
                    E                  E
                                                                                                                                                                                                       field.   As
                                                                                                                                                                    a uniform             electric
                                                                                                                      Torque on a dipole                   in
Clearly, the components of E,and E,, normal to shown in Fig, 1.66(a), consider an electric dipole
will cancel out. The components consisting of charges +qand -4 and of length 2a
                                 =-2.
                                                                                                                       Force exerted             on    charge          -g      by field      E   =-gE
                                                                                                                                                                                                         E)
                                                                                                                                                                                            (opposite to
                                                                                  :cos0=
                                                                                                  J+                                Eotal     =tqE -gE =0.
                                               1
 or                                                                                                                                                                                                  qE
 r>>a, then
                                               1
Fequa
(a)
                                             Fequa
                                                                      1P                                             uniform          electric     field
                                                                                                                                                                                                                                                 to     E.    In   a
                            Torque = Either force                                x    Perpendicular distance                                                                                 is parallel             or arntiparallel
                                                                                                                                                           When     the dipole
                                                                                      between the two forces                                                                                                                                                       to
                                                                                                                                                                            field,       ifp           is parallel        to E or        antiparallel
 distancer                                                                                                                                        non-uniform
e    distance
                                            t=qEx 2a                sin     0 =(qx 2a) E sin 0
                                                                                                                                                   E,the net              torque         on            the dipole is zero (because
                                                                                                                                                                                                                                   the
                                                                                                                                                                                                                        However, there
                                            t=       pEsin         0                                                   (p=qx           2    a)    forces on charges      q become        t                     linear).
                                                                                                                                                                                                                                                           when
                                                                                                                                                       net force on thedipole.    shown in Fig.              As                                 1.67,
                                                                                                                                                  is a
                            As         the direction of torque                                       is   perpendicular                    to                                                                                 on   the dipole            in     the
NIFORM                                                                                                                                            p    is     parallel     to       E,   a       net force acts
                                                                                                                                                                                                                                                      E.
an     electric
                                                                                                                                                  a    net force acts             in     the direction of decreasing
ence     define                                                                                                                                                                                         E
                             The        direction of vector                           t is        that       in    which         a right
                                                                                                                                                                   Force     on     -q
c field. As              handed screw would                            advance               when         rotated          from     p      to
                                                                                                                                                                                                                                           on    -q
Pposite
               to   E)
                                                  Tmax           pE      sin     90°    =pE.                                                                         on+q
                                                          T=      pE sin                                                                                                                                      (b)
     +gE
                                 If    E=1           unit,   0 =90°,           thent=p
                                                                                                                                                                Fig. 1.67       Forces            on a      dipole     (a)    when     p   is   parallel
acting on an electric dipole, placed perpendicular to a uniform to E and (b) when p is antiparallel to E.
1.27 DIPOLE INA NON-UNIFORM of paper. As the comb runs through hair, it acquires
                                                                                                                                                                                                                                piece of paper, it
                                 What happens when an electric dipole is held in
                                 41.
                                                                                    a                                                                 ougt            eloser           to        an uncharged
                                                                                                                                                                                                        paper                 induces a net dipole
                                                                                                                                                      POlarises the Plece of                                          i.e.,
                                                     ? What will be the force and the
                          non-uniform electric field                                                                                                  moment          in    the direction of the                              field.     But     the     electric
                                                                                                              or anti-parallel               to
                           torque        whenthe                dipole      is    held       parallel
                                                                                                                                                      field     due to      the        comb on the                    piece of paper              is    not uni
                                                                ? Hence           explain            why       does a comb                 run
                          the     electric           field
                                                                                                                                                      form.It exerts a force in the direction of increasing                                                     field
                           through dryhair                       attract         pieces          ofpaper.
                                                                                                                                                      i.e.,   the piece of paper gets attracted towards the comb.
                                      Dipole         in     a   non-uniformn                  electric            field.    In    a non
uniform electric field, the + qand -qcharges of a dipole 42. Give the physical significance of electric dipoles.
experience different forces (not equal and opposite) at Physical significance of electric dipoles. Electric
                           slightly             different         positions in the                    field       and hence            a   net        dipoles have a    common occurrence in nature. A
    electric   field.                                     on the dipole               in         non-uniform               field.      Also,          molecule  consisting of positive and negative ions is an
                           force                acts                                         a
n
                                        F
        Torque on
        perpendicular
                               a    dipole is
                                    to       the field E
                                                          maximum when it is                            held          experiences
                                                                                                                      4on, deternine
                                                                                                                                           a   torque
                                                                                                                                                   the
                                                                                                                                                           of8V3 Nn.If the
                                                                                                                                                           magnitude               of     either
                                                                                                                                                                                                          length     of the dipole
        non zero force and non zero torque. In the special case
                                                                                                                                                         2a-4cm                   =       0.04 m                                               positioe      th)a point
                                                                                                                                                                                                                                                          charge,         15                O
        when the dipole moment is parallel or antiparallel to
                                                                                                                                                                                                                                                passng through O and rormal
        the field, the dipole experiences  a zero torque   and a                                                             As      tpE          sin    9 = qx2ax                    Esin        9
                                    an                                                                                                      (2a)E        sin    0
       speifically                                                                                                                                                                                                                                                  Za-5mm-5x
                            E-field
       increasing                                                                                                                                           8/3                                                                                                           r        15cm-15
       may    be     represen
       ted   by    field       lines                                                          Direction         of
                                                                                                                                           0.04    x 4x        10 x sin60°                                                                          (a)Field at the                acial     point
       as shown.                                                                                 increasing                                                                                                                                                                          2p               2
                                                                                                   E-field                                     V3x    10x2 -10'C=1
        Clearly,            E<Eg < Ec                                                                                                             2x v3
                                                                                                                                                                                                            mC.
        The direction              of   the electric field at an axial point of an
        electric    dipole           is      same        as that     of     its   dipole         moment               Example 51. An                       electric         dipole             consists          of tuo         opposite
                                                                                                                                                                                                                                                                                   9x10x2
        and at an equatorial                        point     it     is    opposite          to    that    of         charges of magnitude                     1/3 x 10                   C,     separated            by2 cm.        The                                                          (15
        dipole moment.                                                                                                dipole    is   placed in an               external          field          of        3x 10      NC. What
        The strength of electric                     field   at an axial point of a short                             maximum          torque does the electric field exert on the dipole                                             ?                                       -2.66              10
        dipole     is       twice the strength                at the same distance     on
        the equatorial              line.                                                                                    Solution. Here q                   =, 107C x                        2a=2 cm                  =0.02      m,               This field is directed alo
                                                                                                                                                                 3                                                                              moment          vector,Le.            rom-              to
        At   larger           distances,              the    dipole           field         (E « 1/r')
        decreases           more          rapidly         than       the electric field of a                                                               E=3x              10       NC1                                                                                                         B
        point charge (E                      l/r).
                                                                                                                             Tmav     =pE         sin    90° = qx           2ax Ex                    1                                                         -10 uC
Examples based on                                                                                                                          1
                                                                                                                                                                                                                                                                                                 Fig.
  Dipole Moment, Dipole Field and                                                                                                              x 10         x 0.02 x          3 x 10 x1                         = 0.02 Nm.
 Torque           ona Dipole                                                                                                                                                                                                                          (b) Field at the               equatoria
                                                                 1
                                                                                  2pr                                                                       1
                                                                                                                                                                                  2qa                                                                                          = 1.33        x    10
                                              Exial
                                                                                                                                                                    (+a'2
          When          r >> a,                               1            2p
                                                                                                                                                          4nE
                                                                                                                                                                                                                                                                  dipole
                                                                                                                                                                                                                                                                         the       direction
                                                                                                                                                                                                                                                                                    moment
        from the centre of                     the dipole            is                                                                             =     4.096     x       10     NC                                                             vector,        Le., tromm        +q to-4
                                                                 1
                                                                                   P                                                                                                                                                              as shown          in    Fig.      L69.
                                                                                                                     Example 53.Tuo point                           charges,               each            of   5uC       but   opposite
                                                                                                                     insign,         are placed 4 cm                apart.            Caleulate                  the electric        field
         When        r      >>4,                                                                                     intensity       at a point distant                  4    cm          from the midpoint on                         the
                                                                                                                     axial    line    of   the dipole.                                                                     [Punjab02]
  4.     Torque,t           =pE         sin    @   where 0is the angle between
                                                                                                                          Solution.            Here q -5                    10°C,2a
                                                                                                                                                                        x                                  =0.04      m,
         P   and        E
                                                                                                                                                            a =0.02               m, r=0.04                     m
                                                                                   ELECTRIC          CHARGES AND               FIELD                                                                  143
                                                        2 pr                 1    2(qx 2a)r                   Example 55. Tuo                     identical         electric     dipoles        are   placed
              -10 uC                       +10 uC
                                             Fig. 1.68
                                                                                                                                            =V2 kq,parallel to DC.
       (b)   Field    at the equatorial point                            Q of the dipole is                                         Eyne    = E t E, =2V2kq
                                                         9x2a3                                                                                   2424          parallel    to     ABor      DC.
                                   41
                                                            10C                                + 10uC
                                                                                                              and     due     to       DC is p,,as shown                   in   Fig. 1.171(b).
                                                                          Fig. 1.69
                                                                                          PHYSICS X1
                                                                                                                                                                                                  m.
                                                                                                                 Hereg-2x          10C, 2a 3em-3                                      10
                                                                                                         2.
                                                                                                                           E-2 x   10           NC
                                                                                                                                                              (0.1)
 Electric          field    at   O due to p               is
                                                                                                                                 -3.6       x       10        NC
                                   1
                                                                                                                                                                                             0.
                                                                                                        4.      Here
                                                                                                                                                                                    Cm
 Electric           field   at   O due to p, is                                                                            p=qx 2a = 10×0.10 =10
                     E,     -2,                   along         OD
                                                      1        4
  Clearly,           E -E,-E-2
                                                                                                                                                                          20 cm
  Magnitude                of the net      field      at the origin            O is,                                             20 cm
                    E =VE+               E²     =N2E=
                                                                     1     2V2q
          making an angle of 30° with the direction                                     of   the               (+a?j2 =            20cm              = 0.20 m
          field. Determine the torque exerted by the                                 electric                                                                 P
          field on the dipole.            (Ans. 2 x                                10 Nm)                                Exqua
                                                                                           E
                                                                               R
Fig. 1.73
equilibrium.
imaginary curves. Yet the concept of lines of force is                                                            6.     The       lines        of   force        have        a   tendency          to     contract
very useful. Michael Faraday gave simple explana                                                                         lengthwise.             This explains                attraction          between           two
tions for         many             of his discoveries                      (in      electricity          and             unlike charges.
magnetism)             in terms of such                           lines   of   force.
                                                                                                                          The     lines         of   force        have a tendency to expand
                                                                                                                   7.
                                                                                                                         laterally       so as       to exert      a lateral pressure on neigh
                         For Your Knowledge                                                                              bouring ines of force. This explains                                            repulsion
                                                                                                                         between two similar charges.
         The lines         of   force are imaginary curves,                              but the       field
                                                                                                                   8.    The     relative        closeness          of    the      lines    of    force       gives a
         which they represent                      is real.
                                                                                                                         measure         of     the strength             of   the electric          field      in any
         The term       'lines           of force       is    misleading.           It   will   be more                  region.       The      lines    of force         are
         appropriate
         lines'.
                                to call        them         electric      (or magnetic)              field
                                                                                                                          ()     close        together       in    a strong field.
                                                                                                                         (i)     far   apart in a        weak field.
         A    field    line        is   a space curve                  i.e.,   a curve          in   three
                                                                                                                         (ii)    parallel       and equallyspaced                     in   a uniform           field.
         dimensions.
                                                                                                                  9. The         lines    of force       do not pass through a conductor
                                                                                                                         because the      electric                       field       inside         a         charged
                                                                                                                         conductor is zero.
Properties of              Electric           Lines of Force
   1.     The      lines        of force are continuous smooth                                       curves 1.29 ELECTRIC FIELD LINES FOR DIFFERENT
          without any breaks.                                                                                             CHARGED                    CONDUCTORS
             The lines        of    force         start      at    positive        charges and end
    2.                                                                                                             44. Sketch and explain                         the field lines            of (i)      a positive
                                                                  they cannot form
                                                       -
                                                                                                                        cge ) a
                                                                                                     closed
          at      negative          charges                                                                                                                         point charge,             (iii)     two eaual
                                         is a      single charge, then                     the lines         of Pot                               negatrve
         loops. If there                                                                                       and opposite         charges,          (iv)    two equal positive                   charges and
         force will           start      or end         at infinity.
                                                                                                                (o)a    positively        charged plane conductor.
    3.    The tangent to                 a line        of    force     at any pointgives the                       Electric        field       lines for different                  charge systems :
          direction           ofthe electricfield                      atthat       point.
                                                                                                                   (0)   Field      lines        of a positive point charge. Fig.1.74
    4.       No   twolines              of   force     can cross each                other.                    shows       the lines           of force of an                 isolated        positive          point
                                                                                                               charge. They are directed radially                                  outwards           because a
         Reason.         If     they         intersect,            then there will be two                       small     positive            charge         would            be accelerated                   in   the
                                                                            (Fig. 1.73) and
          tangents         at the point                ofintersection                                          outward          direction.           They extend              to    infinity.      The field is
                                                                   the electric          field    at    the
          hence       two           directions               of
                                                                                                               spherically symmetric     i.e,, it looks                                          same in all
          same        point,       which          is   not possible.                                           directions, as seen from the point charge.
 I.46                                                                                 PHYSICSx                                                                                                                                                                                           ELECTRICC
field es
positive point charge. negative point charge. Crk AQat P and an element of area
of a positive                point charge, the                 electric     field    of a           (o)    Field            lines        of a positivcly charged plane
negative point charge                     is   also spherically       symmetric but          conductor. Fig,                 1.78        shows the pattern                   of   lines     of     force
                                                                                                                                                                                                                                                          OP
the lines        of    force point radially                 inwards as        shown in        of         charged plane conductor. A small positive
                                                                                                   positively
Fig. 1.75.      They start            from       infinity.
                                                                                             charge would tend to move normally away from the
       ()     Field       teo equal and opposite point
                            lines    of                                                      plane conductor. Thus the                               lines       of    force       are parallel
charges. Fig. 1.76 showsthe electric lines of force of an                                    and normal                to    the surface of the conductor. They                                      are
electric dipole i.c., a system of two equal and opposite                                                              that electric                                   field       E   is    uniform
                                   a small distance They                                     equispaced, indicating
pont charges (tq)separated by                                                                at all points near the plane conductor.
start from the positive charge and end on the negative                                                                                                                                                                                                          Fig.   L80
charge. The lines of force seem to contract lengthwise
as    if   the two charges are being pulled                           together.      This                                                                                                                                     Number                 of    lines       of force        cutting      unit
                                                                                                                                     plane      conductor.
                                                                                                                                                                                                                                          E
                                                                                                    45. What           is the relation            between the               density        of lines      of
                                                                                             diagram.                                                                                                                                      E
                                                                                                    Relationbetween                       electric      field    strengthand density
                                                                                             of lines       of    force.       Electric          field    strength    is proportional
                                                                                                                                                                               field       strength
                                                                                                                                                                                                               130 AREA                    VECTOR
                                                                                             to    the density of lines of force                         i.e.,   electric
                                                                                             at    a point       is   proportional to the number of lines of force                                                    47. What             is   an area            vector? How              do   we speci
                                                                                             cutting       a   unit area    element placed normal to the field at                                              direction            of    a planar         area vector              ow do we         asso
                                                                                             that    point.       As     illustrated            in Fig.       1.79,   the electric           field at           vector       to    the area of            a curved           surface    ?
                                                                                             Pis stronger than                      at    Q.                                                                           Area         vector.           We        come          across        many      situ
                Fig. 1.76          Field lines of     an   electric dipole.
                                                                                                                                               Region    of                                                     where we                 need to know not only the magitud
                                                                                                                                              weak   field
                                                                                                                                                                                                                surface area but                      also       its   direction The              directie
      (iv)   Field         lines  of two equal and positive point
charges. Fig, 1.77               showsthe lines of force of two equal                                                                                                                                           planar area vector                   s    specifed      by the      noralto         the pl
and positive point charges. They seem to exert a lateral Fig 1.81(a), a planar area element dS has been
pressure as           if   the     two charges         are being pushed             away                                                                                                                        sented            by a    normal vector dS. The length                              of   ve
from each          other.      This explains repulsion between                       two
                                                                                                                                                                                                                represents the magnitude dS                                  of   the area element.
like charges. The field                   É    is   zero at the middle point             N                              -Region          of
                                                                                                                                                                                                                a unit vector along                       the normal to the planar area,
 of   the join of      two charges. This point is called neutral                                                        strong       field
              P
                      is constant.
         at distances r and
                                  Consider two points
                               from the charge q. The
 same number of lines (say n) cut an
                                      element of area
                                                             ,                                                                                    (a)                                                         (b)
       AQ     at   P and         an element of area                        AN at       Py.                                          Fig. 1.81        (a)    A planar          area       element.       (b)      An     area
                                                                                                                                                     element         of   a   curved       surface.
                                                                   T;=OP,
                                                                                                                          In    a curved surface,we can imagine it to be
                                                                                                                                case        of
represents the magnitude dS of the area element. If n is Fig. 1.82 Electric flux through normal area.
field
E cos
                 A
                    ,
                    E,   then the component
                         so that the electric
                                  Normal component
                                                            of
                                                          flux
                                                                  E
                                                                  is
                                                                 of
                                                                       normal
                                                                       Ex
                                                                                   to
                                                                            Surface area
                                                                                        AS   will   be
                                                                                                                 Electric
                                                                                                                     Unit of
                                                                                                                                 flux   is
                                                                                                                                  dUnit of Ex
                                                                                                                    St unit of electric
                                                                                                                                               ascalar
                                                                                                                                                    flux
                                                                                                                                                         quantity.
                                                                                                                                                                unit of S
                                                                               AreaAS                                                     -   Vmm'Vm.
                                                                                                          1.32 GAUSS'S THEOREM
                                                                                                                 49.State and prove                 Gauss'stheorem.
                                                                                   0                         Gauss's theorem. This theorem
                                                                            Ecos                                                              gives a relationshin
                                                                                                          between the total flux
                                                                                                                                    passing through any
                                                                                                                                                              closed
                                                                                                          surfaceand the net charge
                                                                                                                                     enclosed within the surface
                                                                                                             Gauss theorem states that the total
                                                                                                                                                                               flux    throuoh     a
                                                                                                          closed    surface is 1/ E, times the net charge
                                                                                                                                                          enclosed                          by he
                                                                                                          closed    surface.
                          Fig. 1.83        Flux through     an inclined     area.                                  Mathematically,it can be expressed as
        In case the field
                           E is
                                non-uniform, we consider a
    closed surface S lying
                             inside the field, as
    Fig. 1.84.           We
                  can divide the
                                                   shown in
                                   surface S into small area
                                                                                                                 Proof.For the sake of
    elements :AS, AS,, A,e..,                                                                                                          simplicity,we prove Gauss's
                                                      AS.
                                      Let the corresponding                                              theorem      for an isolated  positive point charge             4. As
   electric         fields        at these elements
                                                                                                         shown in         Fig.    1.85,   suppose the surface S is a sphere of
                                                               be E, E, ,.,EN:                           radius       r   centred       on     q.   Then      surface     S is     a
                                  Closed    surface   S                                                  surface.
                                                                                                                                                                                        Gaussian
=1=0 ds
          The        charge         q       appearing                 in       the     Gauss's           theorem                                Fig. 1.86   Applying            Gauss's     theorem to a
        includes            the    sum of          all       the charges located                    anywhere                                                       point       charge.
        inside           the closed           surface.
The electric field E appearing in Gauss's theorem is By symmetry,E has same magnitude at all points
        due to            all    the    charges,                both       inside       and        outside             the     on S. Also E and d                       at    any     point      on S      are directed
        closed        However, the charge q appearing in
                          surface.
the theorem is only contained within the closed radially outward. Hence flux through area d is
surface.
                                                                                                                                                            9
 133 GAUSSIAN SURFACE                                                                                                                                       1
                                                                                                                               or          Ex     4
        Gaussian
enclosing            a    charge
                                surface.
                                        is
                                                   Any
                                              called
                                                                 hypothetical
                                                                the    Gaussian
                                                                                               closed
                                                                                              surface
                                                                                                              surface
                                                                                                                  of   that
                                                                                                                               or                      E=      1
                                                                                                                                                                            9
charge.      It is chosen to evaluate the surface integral                                                                of
the    electric           field   produced                   by the charge enclosed                                by it,          The force on the point                     charge 4, if placed on surface
 which,      in turn, gives                   the      total      flux         through the               surface.
                                                                                                                               S will be
                                                                                                                                                                               1      49%
       Importance. By                           choosing   a   closed surface                                                                          F=4,        E    =
  E    has   a   normal             component which                              is   either zero or has                             This proves the Coulomb's                          law.
    I,50
                                                                                       PHYSICS-      XI
   Examples based on
    Electric Flux
                  and Gauss's
                                                          Theorem                               Example59.A
     Formulae Used                                                                                                                cylinder      is
                                                                                                    E with
                                                                                                                 its
                                                                                                                                                     placed    in     a uniform
                                                                                                                         axis                                                       electricfield
      1.                                                                                                                          parallel      to the
            Electric    flux                                                                    electric flux                                                ficld.
                                through        aplane surface                                                            through
                                                                                                                                                                         Show    that         the
                                                                                                                                        the                                                         total
            a unifomm                                                  area Sheld
                                                                                       in            Solution. The
                                                                                                                                              cylinder        is   zero.
                             electric      field    E is                                                                            situation          is
                                                                                                                                                            shown in           Fig. 1.87.
      3. Flux
                     density       = Total flux
                                                                                                                left     plane
                                          Area                                                                                          right plane
                                                                                                                       face                                        Curved
     Units     Used                                                                                                                          face
                                                                                                                                                                   Surface
                                                                                                               -|E dS
            Electric
              NC
                        flux       o- is in
                                                 NmC         and
                                                                                                                                cos 180°      +EdS cos                 0°+     E dS      cos 90°
                                                                     flux density
                                                                                       in                      =-E[ds+ E [dS +0
     Constant Used                                                                                         =-Ex T +                      Ex           =0.
             Permittivity
                                   constant of free
                                                    space is
                                                                                              Example 60.Calculate    the
                                                                                                                          number of electric
                                                                                               originating
                                                                                                         from a charge of 1 C.               lines offorce
                                               =8.85x 10-12
                       4x9x 10-9                                     c²   N-m-2                   Solution.The
                                                                                                                 numberof lines of
                                                                                               from a charge of
                                                                                                                                  1C  force
                                                                                                                                                                                 originating
                                                                                                                                                                               Nm²
                                                                                                                                         8.85 x      10-122x             10°
                             = 120    units.                                                                                       Total      flux
                                                                                                    Flux       density
Example 58. A                 circular                                                                                                  Area
                                            plane sheet of radius
placed in a uniform                                               10 cm is
                    electric               field of5 x 10
                                                          NC-, making an                                                            2x 106
angle of60° with the field.
                             Calculate                    electric flux                                                            4n   (0.5)?
                                                                                                                                                     =6.4 x         10    NC,
 sheet.                                                                     through the
                                                 Nm2c1.
                                                                                                                                  19
                                                                                                                                  6   E0
                                                                           ELECTRIC    CHARGES AND                FIELD                                                                       151
   (b) When charge q is placed at one vertex, the flux                                                                                                                   charge inside the
                                                                                                     (ii) By         Gauss's theorem, the                      total
through each of the three faces meeting at this vertex                                        cube        is
        be                                                                                                                                   1
will          zero,      as E      is parallel          to    these     faces.    As only                        9=En             =4nx9x 10 x1.05 =9.27                                  10     12c.
one-eighth          of     the    flux    emerging              from       the charge g
passes through the remaining          three faces                           of the cube,      Exomple (64.)An electric feld is                                      uniform, and                in    the
so the      flux    through each such face is
                                                                                              positive x direction for positine x and                              uniform       ith          the   same
                                 11       4         1    9                                                                                                                                     z
                              3   8             24       &o
                                                                                              magnitude in the
                                                                                              given that                     E=
                                                                                                                                  negative
                                                                                                                                    200
                                                                                                                                                 x
                                                                                                                                                 NC
                                                                                                                                                      direction        for negatie                   It   is
                                                                                                                                                                 for   x >0
    Lample63. The                clectric fieldcomponents in Fig.                  1.88 are
                                                                                              and                            E=-200                   NC-        for     x   <0.
E,   =ax,E, = E, =0,                     in   which       a     =800       N/   CHf.Calcu
        flux through
late (i) the                             the cube and (i) the charge within                          A    right      circular       cylinderof          length20 cm and radius                      5cm
       Assume
the cube.         a=        that          0.1   m                                             has    its       centre    at the origin           and     its     axis along the I-xis so
                                                                                   [NCERT]
                                                                                              that    onefaceis at x               =+10 cmand                the other is at x           =-10        cmt
                                                                                                                     5cm
    Z
                                         Fig. 1.88
     Solution.           () The                                                                 AS
                                      electric          field     is   acting only       in
X=-10 cm I=10 m
A is
these faces.
            +n/2. So flux           =faces,      the angle between
                                                                                                     Solution.               ()    On the
                                                                                                                                             ig.
                                                                                                                                             left face
                                                                                                                                                      1.89
                                                                                                                                                             : E =-200                    NC,
     The magnitude               of the                              at the left face is
                                           electric       field
                                                                                                                             AS   =-AS i=-(0.05i m²
                      E,   =r?=a a?                          x=a at the left          face]
                                                                                                     The outward                  flux      through the            left      face   is
                                                                                                                        AŠ        = AS =r(0.05)i
     Flux,
                      S =Ep AS cos 0° =E,                        a
                                                                                                                                                                         m
[0 =0° for the right face) The outward flux through the right face is
(i)Net outward flux through the cylinder, Example 66. Figure 191 shos five charged lumps of
6,-157 4 L57 +0 -3.14 Nm? C!. plastic and an clectrically neutral coin The cross-section ofa
                 By
                                                                                                                  Gaussian            surface        S   is   indicated          What is      the net electric     flu
         (io)           Gauss's theorem, the                            net     charge inside the
                                                                                                                  through            the surface          if
   cvlinder
                                                                                                                                9,--3.1nC?
                                                                                                                                                     +3.1       n    C,           29%-5.9nC
                                          are given a charge                  + Qat       the origin       O
  (Refer to        Fig. 1.90).            Considera sphere               S with centre          (2, 0,    0)
  of    radius     v2 m.          Consider another                     sphere        of   radius     v2    m
  centered       at the origin.              Consider the spherical caps (i) PSQ                                                                                                 19
  (0) PRQ        (in)     PWO,              with normals outward tothe respective
  spheres,       and (iv)the               flat   circle      PTQ      with normal            along    the
  A-4X1S.
surfaces ()-(i0) ?
                                                  1TR                            W
                                                           (2, 0, 0)                                                                                 +3.1x          10-5.9x 10-9-3.1x                     10-9
                         (0, 0, 0)
                                                                                                                                                                              8.85x 10-12
                                                                                                                                              =-666.67                Nm² C-l,
                                                                                                            Example 67.S,                     and      S, are        two concentric       spheres      enclosing
                                              Fig. 1.90
                                                                                                               charges       Q   and        2Q respectively                as shown      in    Fig.   1.92.
     Solution.For the charge + Qsituated                                                        O, the
                                                                                at origin
                                                                                                                   () What             is    the     ratio      of the
field   E points along +vex-direction                                   i.e.,   towards       right.                      electric      flux through                  S,
                                                                                                                                                                                                   2Q
   (a)    The outward drawn normal on cap PSQ points
                                                                                                                          and        S, ?
          towards          left       while           it   points towards right for                               ()How                will        the        electric
caps PRQ PWQ and circlePTQ.So the flux is flux through the sphere
         negative for                 (i) and         positive for the rest.                                              S,     change,     a medium
                                                                                                                                                if
                 4=EA
                         -4.5         x   10 QA NC'm?.                                                           Flux through                   S,   is              =2Q+Q_3Q
                                                                                     ELECTRICCHARGES AND                         FIELD                                                                 1.53
Ratio of electric flux through S, and S, is 7. A charge q is situated at the centre of an imaginary
                                       Q/13                 =1:3
                                                                                                                          hemispherical         as shown in Fig, 1.93. Using
                                                                                                                                                      surface,
                                                                                           1        Q
                                                                                           K        En
             =8í+
             E              4+           3k NC-,            calculate              the     electric        flux
                                                                                                                          where E is            in    NC     and       x is    in    metres.
                                                                                                                          Find
                                uniform                          field        É    =3x10° i              NC-1,
 3Consider                  a                     electric
                                                                                                                           (i)   net flux         through the           cylinder,
                                                                              through               a    square
        Calculate the                   flux      of   this      field
                                                                                                                          (i) charge enclosed by the                           cylinder.       [CBSE    D 13]
        surface of area 10                      cm when
         () its         plane          is parallel         to   the y-z plane, and                                  9.     The electric field in a region                      is   given by   E   =b
                                                                  makes a            60° angle with
        (ii)      the   normal to its                  plane
                                                                                                                          Find the charge contained in the cubical  volume
                                                                                      [CBSE D 13C]
                  the x-axis.
                                                                                                                          bounded   by the surfaces x      0,  x =4, y=0               =
                                                        30 Nm            c-         (ii)   15Nmc-']                                                      E, =5x 10 NC
                                        [Ans. (i)                                                                          y= a, z    =0
                                                                                                                                      and z=a. Take
                                                    (ii)25 Nm c-')
                              [Ans.(i) 50 Nm'c-
                                                                 of a
                                           located at the centre
         A point        charge of 17.7uCis
                                                                                         flux       through
                              Find the                                 electric
        cube of side 0.03 m.                                                             [Himachal            931
        each face of the cube.
                                                              (Ans. 3.3 x 10°
                                                                                               Nmc-)
                                                                                           a    charge         of
            Calculate () the flux                      through the cube,and                      ()the         8.   (0       Flux through the curved surface                              of   the cylinder
                                                                               ICBSE18C;SP 20)                                    zero.
                                 ,
                                                                                                                             is
            charge    inside         the cube.
|Ans (0 -01 Nm c'()4-8854 x10C Magnitude of the electric field at the left face
                                                                                                                                      E-50x 1-50NC
 11.         Auniform           electric field                 E           E,i N/Cor x           >   0and
                                                                                                                                      ,    EScos                50x 25     x10cos 180
            E--E,iN/Chor x <0are given.                                              A   right circular                                                   10Nm
                                                                                                                                          --1250
            cvlinder of length                  /cm and radius                    r cm has    its    centre
                                                                                                                             Magnitude          of   the electric field                atthe     right      face
            at   the oigin and             its      axis       along the             -axis.   Find out
            the    net   outward           flux.        Using Gauss's law                     write     the                           E= 50x2= 100NC
            expression           for   the net charge within the cylinder.                                                                 100×25x 10cos 0
HINTS
                                                                                         |CBSE      D 08C]
                                                                                                                                       - 2500x           10Nm!
                                                                                                                                  o    =t            p-(2500 - 1250) ×10Nm'c
    1.E -8i+4j+ 3k
                                                                                                                                       - 1.250x 10
                                                                                                                                                     10Nmc!
                                                                                                                                      charge enclosed by the
                                                                                                                                      9-0r           =    8.854
                                                                                                                                                                   Nm
                                                                                                                                                                   x
                                                                                                                                                                           c1
                                                                                                                                                                       101250 x10c
                                                                                                                                                                                    cylinder,
                         ,
                     the electric field,
                                                                                                                              a E          5x 10°          x(0.01
                           o     = ES cos @ = 3x 10°                       x(0.10)cos       0°                                                                             =   0.25      Nm²C
                                                                                                                                  b                  0.02
                                 - 30 Nmc-1
                                                                                                                     q=                =8.85         10x0.25                   -   2.2    x 1O    12   C.
                 ()              =3x       1o       x(0.10)           xcos 60°       = 15 Nm²c-1.
                                                                                                                                                 x
                                                                                                                                           YA
       4.        ()      =     EScos
=5×10°x(0.10)x=25 Nm²c1,
                      = 3.3      x   10 Nm? C1
                                              8.85         x   10C
 6.         () =                          x     10-12          C²    Nm2                                                                             Fig. 1.96
                      Eg         8.85
                                                                                   through          entire
                                                                                                                             =[E,(at x = 2a)-
                                                                                                                             =[a(20)-
                                                                                                                              aa'     = 400
                                                                                                                                               a(a)] a
                                                                                                                                                x(0.1)
                                                                                                                                                          E,    (atx =
                                                                                                                                                                = 0.4
                                                                                                                                                                           a))
                                                                                                                                                                         Nm²c
                                                                                                                                                                                   a
  7.
        spherical          surface        is                                                                             9             =   8.854         x10x0.4               = 3.542         x10C.
                                                                                                              11,   Proceed as in Example 64                           on page           151.
                                                       14                                                                             -   2r E,          (10)      Nmcl
                                                       2
                                                                                                                      ()q6,E=2ur'E, (10)c
                                                                             ELECTRIC            CHARGES AND                   FIELD
                                                                                                                                                                                                     155
135          FIELD DUE                  TO AN         INFINITELY                                            Vectorially,
                LONG CHARGED                         WIRE
       52.      Apply Gauss's             theorem           to   calculate            the electric      whereî             is   the     radial         unit vector in plane normal                       to   the
fcld    ofa thin      long straiglht line
                         infinitely                                         of   charge,with        a
                                                                                                        wire passing through the observation                                         point.
uniform charge density of à Cm
       Electric      field   due to an         infinitely        long      straight      charged        1.36 ELECTRIC                            FIELD         DUE TO A UNIFORMLY
wire.    Consider a thin infinitely long                          straight        wire having                        CHARGED                      INFINITE PLANE SHEET
 auniform     linear charge density 2                            Cm         .By symmetry,                   53. Apply                   Gauss's theorem                  to   calculate        the electric
the field E of the line charge is directed radially outwards field due to an infinite plane sheet of charge.
from the line charge. To determine the field at a plane sheet. As shown in Fig. 1.98, consider a thiun,
distance rfrom the line charge,                             we choose             a   cylindrical       infinite          plane sheet of charge with                                uniform          surface
Gaussian surface of radius                          r,   length        l   and with        its   axis   charge density G.                       We wish          to calculate          its electric       field
along the line charge. As shown in Fig. 1.97(), it has                                                                         P at     distance r from                 it.
                                                                                                        at a point
Curved surface S, and flat circular ends S, and S,:
                                                                                                                                                                              Plane sheet,
Obviously, ds,||E, ds,IE and ds, 1 Ë .So only the charge density
                                                                                                                Cross-sectional
                                                                                                                         area    A
                                  S
                                                                                                                                             =EA+           EA     =2EA
                                                                                                                                                                                                     q   =GA
                                                                                                                Charge enclosed by the Gaussian                                     surface,
                                                                                       cos 90°
                                                                 90°+|EdS,
         -Eds,cos 0° + EdS,                              cos
                                                                                                                According                  to   Gauss's theorem,
                                           S
         = E[ds, +0+0
         =Ex area thecurved   of
                                                          surface or              -=Ex 2          rrl
                                                                                                                                                 oA
                                                                                            =l
         Charge enclosed
                                                            =q/ E
                                                                           surface, q
                                                                             We get
                                                                                                                                2 EA=
                                                                                                                                             E   =°
                                                                                                                                                            or          E=
                                                                                                                                                                               28
 Or              E.2n rl      =                or           E=
                                                                    2T eg
                                                                                                                Vectorially,
                                                                                                                                                  2     E
   O      to
                                                        r from  the  line charge.
                           of E        with distance
 the     variatíon
1,56                                                                                    PHYSICS-XI|
   Clearly, E is independent of                         r,   the distance from the                  55.     Two     infinite       parallel     planes have uniform charge
plane sheet.                                                                                   densities      to. Determine             the electric
                the sheet is posilively
                                                                                                                                                               field in      ()the region
    (0)    If                                     charged (a              >0),   the   field    belween the planes, and                 (ii)    outside        it.
           is   directed away from               it.
                                                                                                    Electric        field    of two            oppositely
   (ii) If      the sheet is negatively                                                                                                                                charged plane
                                                   charged             (o <0), the field       parallel      plates.       As shown            in   Fig.       1.100,    consider twe
           is   directed towards it.
                                                                                               plane       parallel       sheets having uniform
                                                                                                                                                                        surface charge
    For a         large planar sheet, the above formula
                 finite
                                                                                               densities       of   t G. Supposer                be a unit vector                     pointing
will be approximately valid in
                                      the middle regions of                                    from    left to      right.
the sheet, away from its edges.
    54. Two infinite parallel
                                 planes have uniform charge                                                                                                                       E
 densities  of o,and G,.
                             Determine the electric field at
points (i) to the left of the sheets,
                                      (ii) between them, and
(ii)to the right of the sheets.
                                                                                                                                    2E0 G,
                                                                                                    Total     field,      E-E         +E,=                                     G      =0
                                                                  E                                                                                   2   E           2eo
                                                                                                   In the region             II   :Fields due to the two sheets are
                                  Sheet   1
                                                                  2
                                                      Sheet
                                                                                                                                          2Eo                           2E
                                          Fig. 1.99
                                                                                                   Total     field,
      In the region I:Fields due to                           the two sheets are                                                         2E               2Eo
                                                                                                   In the region III:Fields                    due
                                   2Eo                            2E
                                                                                                                                                      to       the two sheets are
     From the                              of
                            principle             superposition, the                   total                          E   =2&o
 electric field       at any point of region Iis
                                                                                                                                              Ez2&o
                                                   r                                               Total field,           Em =0.
                          -+E--; 2&0-(o,+ o,)
                                                                                                   Thus       the
                                                                                                              electric  field between two oppositely
      In    theregion II: Fields due to the                            two sheets are          charged plates of equal charge density
                                                                                                                                             is uniform
                                                                                               which is equal to     and is directed from the
                                                                                                                                              positive to
                                  2Eo                          2E0                             the negative plate, while the field
                                                                                                                                   is zero                            on the outside
                                                                                               of the two sheets.     This arrangement
                                   E,                                                                                                                                   is    used
     :.    Total    field,
                                           260
                                                  (o-,)                                        producing uniform electric field.
                                                                                                                                                                                             for
     In the        region III: Fields due                    to   the   two sheets are         1.37       FIELDDUE TO A UNIFORMLY
                                                                                                          CHARGED THIN SPHERICAL SHELL
                              2E                  „20                                              56. Apply              Gauss's theorem                 to    show         that      for    a
                                                                                               spherical     shell,
                                                                                                              the electric field inside the shell vanishes,
                                   Em=
                                              2Eo (o, t o,)
     ..    Total    field,
                                                                                               whereas outside it, the field is as if all the
                                                                                                                                              charge had been
                                                                                               concentrated at the centre.
                                                                       ELECTRIC              CHARGES AND                 FIELD                                                                             1.57
Electric
spherical
                       field
                  shell.
                                   due
                                Consider
                                          to    a uniformly charged thin
                                                 a thin spherical shell of
                                                                                                        or                      E=         9                                                      [For      r= R]
                                                                                                                                        4n E,           R
aroeof radius                  Rwith uniform            surface charge density
                                                                                                                                E=
o.   From symmetry,we see                         that the electric            field       E   at
                                                                                                                                                                          surface
                                                       Gaussian
                                                        surface
                                                                                1      4
         E
                                                                                                                                                                     Spherical       shell,
                                    R                                                                                                                            charge         density    =o
                                                              Spherical       shell,                                  Fig. 1.102     (a) Gaussian               surface         for inside      points
4- =Ex 4y2
     (a)
             Fig. 1.101
             When       point
                               Gaussian
                           a thin
                                            surface
spherical
                                    P   lies
                                                       for outside
shell of charge.
                                                                                    shell.   The
                                                                                                             Applying
                                                                                                                       Ex    4=0
                                                                                                                                ;
                                                                                                                                Gauss's theorem,
                                                                                                                                     =9
total    charge        qinside the Gaussian surface is the charge
on the        shell    of radius R and area                  4R.                                                                 E=0                                                                 [For    r< R]
Flux through the Gaussian surface, ¢: = Ex 4r shell is zero at all points inside the shell.
                                                                                                                                                                                                                       r
     By Gauss's             theorem,
                                                                                                        from the centre of the shell of radius R. E                                              is       zero from
                                                                                                        r=0 to r=R; and beyond r= R, we have
                                                                                                                                                            1
                               =                                                                                                                    E   c
              Ex 4n2
                                    1                                                                                                 E
                                                                                [For r> RJ                                                              E=            1     9
                                                                                                                                                                 4 rE,R*
     Vectorially,               E=4nr
                                                                                                                                                                            E«
        This    field is     thesame as that produced by a                            charge        4
                                                                                      the shell,
 placed at the             centre O. Hence for points outside
                                               is as if the entire
the field due toa uniformly charged shell
                                            centre.
                                                                                                                                            E       =0
                                     at its
 Charge of the shell is concentrated                                                                                                                             R
                                                                          The Gaussian
    (b) When point P lies on the spherical
                                            shell.
1.38 FIELD DUE TO A UNIFORMLY                                                                                 This field         is    same as             that        produced                   by a charge q
                                                                                                       placed       at   the centre             0.     Hence forpoints                   outside             the sphere
     CHARGED INSULATING SPHERE
                                                                                                       the field     de        to     the wnifornly charged sphere                                      is    as   if   the
57. A charge q is uniformly distributed within an entire harge of the sphere is concentrated at its centre.
insulating sphere of radius R. Apply Gauss's theorem to (b) When the point Plies on the sphere. The Gaussian
find the     electrnc       feld due           to this      charge     distribution  at a              surface just encloses the charged                                              sphere. Applying
 pointdistant          rfrom the centre                   the sphere,      where (a)r> R               Gauss's theorem,
                                                    of
(b)    r=R       (c)    0   <r<R Shoo                    the   variation          of   E   with
                                                                                                   r
graphically.
                                                                                                                                      Ex4nR?
            field due to uniformly charged
      Electric
                                              insulating
sphere. Consider an insulating   sphere (For example, a                                                                                                                                                 [For       r= R|
nucleus with protons almost uniformly distributed
inside it) of radius   A with  uniform volume charge
density p. From symmetry, we see that the
                                                                                                             (c)   When         point       P   lies       inside        the       sphere.         The         charge
                                                                                 electric    field     enclosed by the Gaussian surface                                          is
 E    at   any point          is radial         and has          same magnitude                   at
                                                                                                                                        4
points equidistant from the centre                                O   of the sphere           i.e.,                                                                         4
the    field     is spherically           symmetric. To determine                       electric
By Gauss's theorem,
                                                                                                                    Ex 4u =
                       E
                                                                                                       or          Ex 4ny2
                                                                                                       Or                       E=          1
                                                                                                                                                  qr _ pr
                                                                                                                                        4nE,           R          3en
EA
                            =1                                                                                                                  Emax
                                                                                                                                                                  19
             Ex 4nr                                                                                                                                              4 rEG       R
                                          1
                                                                                                                                                                             1
                                                                                                                                                                   E
Or                               1
                                                                                  [For r> RJ
                                                                                                                                                 r=R
Examples based
                  on
 Applications of Gauss's Theorem
 Formulae Used
                                                                                                          charges
                                                                                                          their
                                                                                                                    ,,
                                                                                                          Example 68. Two
                                                                                                                     and              per
                                                                                                                                                  unit length.
                                                                                                                                                the
                                                                                                                                                                          The
                                                                                                                                                                               parallel
separation
                                                                                                                                                                                        force
                                                                                                                                                                                               wires
                                                                                                                                                                                                  exerted
                                                                                                                                                                                                         carry
                                                                                                                                                                                                     between
                                                                                                                                                                                                          on
                                                E=                                                                             E=
                                                     2tE
                                                                                                                                        27e, d
        where       r        is    the       perpendicular distance                       of the                                                                               2 due         to   the   above
                                                                                                               Force per unit length of wire
        observation point from the wire.
                                                                                                          field
  2,    Electric    field          of an         infinite   plane sheet          of    uniform                                                              wire2                                      = Ei,
        surface     charge density o,
                                                                                                                     f=Ex          charge on unit length of
                                                E=                                                        Or
                                                     2Eo                                                             f=;
                                                                                                                          27E, d
  3.    Electric        field       of      two      positively        charged            parallel
                                                                                                          Example         69. An             electric             dipole        consists          of charges
                                                                                                                                                                                      2mm
        plates      with charge                   densities      o,   and o, such             that
                                                                                                          +2 x 10C separated                      by     a distance             of                Itis placed
2Eo 2 cm fron the line charge. Calculate the force acting on the
                    1                                                                                     dipole.
                              (o-o,)                            (Inside the plates)
                  2Eo
  4.    Electric         field             of two         equally and               oppositely
                                                                                                                                                                           -4
                         E=                          (For inside           points)
                                                                                                                                                 2 cm                  -bk-2           mm
  5.     Electric        field         of    a    thin   spherical          shell    of    charge
                                                 .
                     1
                                                     For    r= R       (At the
                                   R                                                  surface)
                                                                            uniform charge
                                                                                                                                              1         22
  6.     Electric       field       of      a solid      sphere       of
                                                                                                                                   E=
         density p and radius R :
                     1                                                                                            Force exerted             by this        field      on charge              q.
           E=                                        For r>         R(Outside points)
                                                                                                                                                          1          2q
                                                                                                                                   F=       qE =
                                                                                                                                                                      r
           E=                                        For    r<R (Insidepoints)
                   4T     E        R3                                                                             Force exerted             on negative              charge (r=0.02 m),
           E=
                     1
                                   4
         Here                                                                                                             =7.2 N, acting towards the                                  line   charge
                                   3                                                                                                                                                 (r =2.2      x    10   m),
                                                                                                                  Force exerted on positive                         charge
 Units Used                                                                                                                   9x      10     x2x2 x               10-x4x10-4
           Here chargesare in coulomb,
                                        r and Rin metre, A                                           in              E,   =                            2.2 x 10-2
          Cmo     in              Cm,
                             pin Cm    and electric field E                                          in
Net force on the dipole. Exomple 72.A charge of 17.7 10Cs dstribulot
                                                                         .
circular        plane perpendicular                     to   the wire.         Deduce          the   exyre
                                                                                                                                                          200               m
 ssion       for its kinetic               energy.     (b) Plot          a   graph      of   the kinetic               Electric         field       at a distance of                          20 cm from                it   in air,
energy as a fnctiot                         of charge      density            [CBSE F 13; OD 23]
                                                                                                                                                                8.85x 1o
     Solution. The electrostatic force exerted by the line                                                                                                                                           -5x 10°          NC!
                                                                                                                                            2Eo           2x8.85                    x   10
 charge on the electron    provides the centripetal  force
 for the revolution of electron.
                                                                                                               Example 73.Two large, thin                                               metal plates are              parallel    and
       Force exerted by                       electric       field       =   Centripetal         force         close to cach other. On their                                            inner faces,            the plates       have
                                                                                                              the    fields       due to           the two plates are equal                                     and opposite.
                                                       or                                                     So,    E =0.
                      2TE                                                     2TEg m
                                                                                                                     (b)     Electric         field       in the region between                                         the     two
       Kineticenergy of the                          electron            will be
                                                                                                              plates,
                                       1                 e
                                                                                                                                                       17.7x 10 22
                                                                                                                                                                                             = 2.0 x 10-10            Nc!
                                                                                                                                              Eg       8.85x 10-12
       (b) As         E      i,the graph                     of
                                                                         1
                                                                         E
 kinetic         energy           E         Ds.    charge
                                                                                                              Example74. A                    large plane sheet ofcharge having surface
                                                                                                                                                                                                of
                                                              Fig. 1.105                                     normal         to the circular             area        makes an angle of 60° with                                   the
 Example 71.                      A long           cylindrical           volume         contains       a
                                                                                                             Z-axis.
 uniformly         distributed               charge of density                 p.   The      radius    of         Given          that   :    e =8.85            x   10-12                 c? N'm2
cylindrical       volume          is   R.A        charge particle (q)revolves                   around
the cylinder          in    a circular         path.    Find      the kinetic       energy        of the
                                                                                                                  Solution.                  Here           g=5.0x 10                           Cm, r=0.1 m,
particle.                                                                     JEE    Main      June 22)      6=60°
      Solution. Charge per unit length of the                                           cylinder,
                                                                                                                 Field          due to        a plane sheet of charge,
                                                                                                                                        E=
                            2-1_pxnR²I =prR?                                                                                                     2E0
                                                                                                                 Flux through
                           E=                        prk' pR?                                                                                  circular          area,
                                                                                                                                             -     4.44     x   10                  Nm²c1.
                      my     =                                                                               Example 75. A                    spherical         conductor of radius 12                                cm     has a
                                  2E                                                                        charge         of     1.6x       10C                distributed                    uniformly over its
                                                                                                            Surface.        What        is    the electric                   field           (i) nside the sphere,
                       K=mu²_pgR?
                         2                                                                                   (ii)just      outside          the sphere,          (H)at                   a point 18 cm from the
                                                                                                            centre     of   the sphere             ?                                                   (NCERT|
                                                                                      ELECTRIC CHARGES AND FIELD                                                                                                      1.61
        Solution.         Here q =1.6 x 107                              C                                        7.    A   spherical       shell of metal has aradius of 0.25 m and
                                          R=12 cm =0.12m                                                                carries      a   charge of0.2 uC. Calculate the electric field
                                                                                                                                           at          a point               (i) inside        the     shell,         (i1)    just
        () Inside       the sphere,               E =0. This             is   because the charge
                                                                                                                        intensity
                on the outer                                                                                            outside the                shell          and        (ii)   3.0m from           the centre                of
resides                                       surface of the spherical conductor.
                                                                                                                        the   shell.                                     [Ans. (i) 0 (i)             2.88       x 10     NC!
        (i)    Iust    outside            the sphere,              r=R=0.12                m.Here         the                                                                                         (ii)      200 NC-)
charge          may be assumed                               to be concentrated                     at the      HINTS
centre of the sphere.
                                                                                                                                                                   Er         9x 10        x   0.04
                           E=                 1
                                                                                                                  1.    2=2rns, Er          =4ne,                  2            9x10 x2
                                          4ne           R?
                                                                                                                          =2         x10           Cml,
                                        9x        10 x       1.6   x 107
                                                                                                                  2.    Here         h=2x10Cm,r= 0.2 m
                                                      (0.12)²
                                                                                                                                                              1         22                          2x2x 10-8
                                    =    10       NC1                                                                   ..E=
                                                                                                                                                                         1
                                                                                                                                                                              =9x 109                    0.2
E= 4
                                    =4.44         x     10     NC-,
                                                                              (0.18)
                                                                                                                  4.    E=9
                                                                                                                              q=&, AS E=                          8.85 x      102 x       1x 100
                                                                                                                                                        (0.25)
                                               mass     5x 10g            is   kept over a large
                                    of                                                                                                     1
   0.       A    particle
                                                    density                            4x 10°Cm.                              E=
           horizontal
                                    sheet of charge                                                                     (rii)
                                                                         to this particle,
                                                                                                    so that
               What charge should
                                   be given
                              does  not fall                                   down.       How many                                      9x10x0.2 x 106                             =200        NC-1
                 released. it
          if
                                                  to give this charge
                                                                       ?                                                                                (3.0)
           electrons           should be removed
                                                             1.355 x 109)
                                              2.16 x 10"C,
                                                      (Ans.
   T62                                                                                   PHYSICSX1
                                                                                                                                                                it.
Repulsion is the surest test of electrification.                              Problem 12 A comb run
                                                                                                                 through one's dry hair
     Problem 6. A positively           charged rod attracts a attracts small bits of paper. Why ? What happens if the
                                                                          hair  is wet or if it is a
suspended object. Can we conclude that the object                                                      rainy day ?
                                                                                         is                                                                 NCERT]
negatively charged               ?                                                                      Solution When the comb runs
                                                                                                                                          through dry hair, it gets
    Solution. No. A                 positively        charged rod can
                                                                                                   charged by friction. The
                                                                                  attract                                         molecules in the paper
                                                                                                                                                                  get
 both    a neutral                                                                                 polarized   by the charged comb,
                      object
                                                                                                                                  e.
                                                    INCERT]
                                                                              4     0.8  10 1"C
     Solution.  Moving vehicle gets charged due to friction.                                      0.5
 The inflammable material may catch fire                                       e    1.6x1019                                                                                 c
                                          due to the spark
 produced by charged vehicle. When metallic rope is
                                                      used,      This is not an integer. So a body cannot have                                                                                                   a   charge
 the charge developed on the vehicle is transferred   to tho of 0.8 x 101"C.
 ground and         so      the fire    is    prevernted.
                                                                                                                  Problem         22. If        the distance between                           two equal point
        Problem         15.    An      inflated           balloon           is    charoed            by    charges      is    doubled and their individual charges are also
 nubbing with fur.Will it stick readily to a conducting                                                    doubled, what would happen                                       to   the force between                   them     ?
 wall or to an insulating wall ? Give reason.
                                                                                          [Roorkee]               Solution.         The original                 force      between the two charges is
      Solution.      It will
                       stick readily  to the conducting wall. It
                                                                                                                                                    1
      Solution.         The charge on                                                                             Problemn             23.    The           electrostatic               force       between               two
                                                      the      fixed     sphere induces
 unlikecharge at the closer end and like                                charee on the far                  Charges      is    a   central           force.           Why ?
 end of the free sphere. Net attraction                                 acts          on the      free            Solution.         The       electrostatic                 force       between two charges
 sphere and so it gets             accelerated         towards the               fixed        sphere.      acts    along the           line joining              the two charges.So it                     is   a central
      Problem        17.      Is   there      some way            of producing high                        force.
 voltage    on your body without getting a shock ?                                                                Problemn        24. How       the Coulomb force between two
                                                                                                                                                         is
      Solution.  Ifwe stand on an insulating surface and                                                   charges      affected          by the presence of a third charge ?
 touch the live wire of a high power supply, a high poten                                                         Solution.            The Coulomb force between two charges
 tial is developed on our body,without causing any shock.
                                                                                                           does not depend on the presence                                        of    a   third       charge.
    Problem 18. A charged rod attracts bits of dry cork                                                       Problem 25. Two equal balls having equal positive
 which after touching the rod, often jump away
                                                trom it                                                    charge 'g coulombs are suspended by two
                                                                                                                                                     insulating
 violently.      Why ?
                                                                                                           strings     of    equal length. What would be the                                            effect       on the
      Solution.     The charged rod                  attracts     the bits of dry cork
                                                                                                           force    when a plastic                      sheet is inserted between the two?
by inducing unlike charge at their near ends and                                                  like
charge at their far ends. When the cork bits touch the                                            rod,
                                                                                                                                                                                                        [CBSE        OD    14]
they share the charge of the rod of the                                same sign              and    so
                                                                                                                  Solution.            The    force           between             the two balls              decreases
get   strongly      repelled        away.                                                                  because      k(Plastic)              >       l   and Foc 1/           K.
electrostatics          ?                                                     ICBSE OD              01CI   a distant d apart in air is F. Ifthese charges are kept at
                                                                                                           the samne distance in water, how does the electric force
                                              Or
                                                                                                           between them change                               ?                                          [CBSE        OD    11]
        Iwocharges 4, and q,,                  separated by             a   small distance
                                                                                                                   Solution.            Dielectric           constant            for    water, K= 80
 Satisfy    the equation 4, +4, = 0.                   What        does          it    tell    about
       Problem 28,Give an example                               to illustrate       that       electro                        Problem          35.        An    electron    and a proton are
static       forces     are   much stronger than gravitational                                 forces.                same      electric          field.         Will   they experience same
                                                                                                                                                                                             keptin
                                                                                                                                                                                                   the
       Solution         A charged                         rod can                                                                                                                                                        force and
                                                glass                  lift   a   piece of paper                  have        same acceleration                         ?
against        the gravitational                   pull of      the    earth      on       this     piece.               Solution.                 Both                           and proton
This shows that the                    clectrostatic              force       on
                                                                                                                                                               electron                                       will exerien
                                                                                   the piece of
 paper is much greater
                                                                                                                  force        of    same
                                                                                                                                    magnitude, F eE Since a proton has
                                           than     the gravitational             force      on     it                                                                                                                             1e34
                                                                                                                      fimes more mass than an electron, so
                                                                                                                                                           its                                           acceleration
       Problem              29,   Two            electrically          charged                                    be
                                                                                                                                                                                                                                       w
charge
situations,
charge on an
               ,
       Solution.
                     electron
                      the
                              or a proton is not
                      Because of charge quantisation,
                              source
                                                      the test
                   cannot go below e. However, in macroscopic
                                        charge is much larger than the
                                                                                   possible              ?
                                                                                                               of a
                                                                                                                       Problemn
                                                                                                               charged conductor
                                                                                                                       Solution.
                                                                                                                        conductor and
                                                                                                                                         40.
                                                                                                                                        This        is
                                                                                                                                                         Why
                                                                                                                                                         ?
                                                                                                                                                                 is    electric
                                                                                                                                                                                                                     surface
                                                                                                                                                                                                                                  a
                                                                                                                                                                            it.
                        electron      or proton,so the limit g, -> 0for the
test   charge         is justified.
                                                                                                                      Problem           41.    Why do the electrostatic                               field      lines      not
                                                                                                                form closed loops?
   Problem 33.What is the advantage of
                                       introducingthe
                                                                                                                                                                                         [CBSE        OD 14, 15;D20]
concept of electric field ?                                                                                            Solution,        Electrostatic                 field lines         start      from a         positive
              By knowing the electrical field at a point, the
       Solution.                                                                                               arge           and end on                 a   negative charge or they                        fade out          at
force on a charge placed at that point can be                                                                  infinity       in    case of isolated                   charges without forming
                                              determined.                                                                                                                                                                  any
                                                                                                               closed
       Problem         34.    How docharges interact ?                                                                    loop.
       Problem         42.    Why do             the    electric      field     lines   never:              Solution.       If   the field lines are not normal, then the field
             each   other?                                           (CbSt     OD A;D 20)
 cross                                                                                               F would         bave         a    langential component which will make
       Solution.       If    two   lines         of    torce     intersect,      then there          electrons      move              alongthe surface creating surface currents
  would       be    two tangents and ttwo                   directions    of    electric field                                                             be
                                                                                                     and the        conductor               will     not        in     equilibrium.
        Proble
                        43.     Draw
                                                                                                     large    metal       plates,       P, and        P,, tightly           held
lines    of force to represent
                               field.
                                                                                                     against     each other and placed between
 auniform electric
                                                                                                     two      equal and unlike point charges
                       [CBSE   OD     05)
                                                                                                     perpendicular               to    the line joining them.                           +0
       Solution.       The    lines     of
                                                                                                            (i)   What            will  happen to               the plates
force    of a uniform electric                                                                                                                                        ?
                                                                                                                  when       they are released
field       are    equidistant                         Fig.   1.108 Uniform      electric   field.
                                                                                                           (ii)   Draw       the pattern of the                      electric
parallel       lines   as    shown      in
                                                                                                                  field     lines for the system.
        1.108.
Fig.                                                                                                                                                            [CBSE       F 09]              Fig. 1.111
       Problem 44. Fig. 1.109           shows electric                        lines of force
                                                                                                            Solution.
due           point charges        4, and q, placed at                   points A and          B
        to
                                                                                                            (i)   When           released,           the two              plates      tend         to    move
                       Write the nature of charge                        on them.                                                                          the charges induced in them.
respectively.
                                                                                                                  apart slightly            due       to
P P
A B
                                                                                                       +Q0
                                         Fig.      1.109
                                            pointing towards 4
    Solution.As the lines of force are
 as wellas q9, so both q,
                          and q, must  be  negative   charges.
                                                (+Q)   is kept in
    Problem 45. A positive point
                                       charge                                                                                                        Fiq. 1.112
                                                           Sketch
                                   conducting plate.
the vicinity of an uncharged                                                                                                                                                 shown in             Fig.   1.113,
                                                       charge  on                                                           48.       In the       electric         field
                      originating from the point
                                                                                                             Problemn
electric field lines                                                                                                                       lines          on    the         lefttwice the
                                                                                                                                                                                    have
                                               [CBSE OD 09, 17C                                      the      electric       field
tothe surface of the plate.                                                                                                                                     those on the right. If the
                                                                                                     separation as that between
                                the charge +2, the lines of
    Solution. Starting from                                                                                                    point A                                      is   40 NC-1, calculate
                                     plate, inducing negative
                                                                                                     magnitude of the field at
 force will terminate at
                          the metal
                                                of force will be                                                                                    proton placed at point A Also
                                                                                                                                                 by a
                        positions, the   lines                                                       the force experienced
         on it. At all                                                                                                                                                 B.
 charge
                                        as shown   in Fig. 1.110.
                                                                                                      find the magnitude                    of electric field at point
                       metal surface,
 perpendicular tothe
                                                      +Q
                                                                                                                                                                                              A
                                                                                                                                       B
Induced
                                                                                                                                                                      point A,
                                                                                                             Solution.       Force on proton                    at
                                                                                                                                                                                                  at    point   B
                                                                                                                                              between the lines of
                                                                                                                                                                                      force
                                                      1.110
                                         Fig.
                                                                            lines
                                                                                                             As the       separation
                                                            that the field                                                that at point               A,   so
                                                                                                           twice
                                                                                                                                                     =x
                                                                                                                     of
     Problem 46. Why is it                        necessary                                           is
                                                                  of a conductor
 from a point charge placed in the
                                   vicinity
                                                                                                                                       - E                      40 =20 NC!.
                                      every point.                                                                               Ey
                       conductor at
       be normal to the                        [CBSE F                                        091
    st
                                                                                                PHYSICS-XIl
              P'roblem         49     The     electric        lines      of   force     tend       to       surface        'S. What is           the       electric
                                                                                                                                                                        flux due
    contract lengthwise                   and expand           laterally.     What do they                                                                                       to
    indicate?
                                                                                                            figuration       through the surface                    'S?                   this
                                                                                                                                                                                                 con-
                                                                                                                                          Net charge
          Solution       The lengthwise contraction       indicates
                                                                                                                                                                 enclosed by the
                                                                                                                                                                                     surface     s
   attraction       between unlike charges while
                                                 lateral expansion
   indicates        repulsionbetween like
                                                              charges.                                                                   +24-44
       TroblenmS A point charge
                                 placed atany point on the
   axis of an electric
                         dipole at some large
                                                  distance                                                        Problem
                                                                                                                        55. Two
   experiences a force F. What will                                                                                              charges of
                                    be the force acting on                                                                                   magnitudes-20 andsn
   the point charge                                                                                     are located at
                     when its distance from the                                                                          points (a,0)and (a,0)
   doubled ?                                     dipole is                                                                                         respectively.  Whas
                                                                                                        is the electric
                                                                                                                        flux due to
                                                                              ICBSE      OD 231                                     thesecharges
                                                                                                                                                   through a sphere of
         Solution         At any                                                                         radius'3a' with its
                                         axial   point of a dipole,                                                            centre at the origin ?
  varics as                                                                   electric     field                                                         (CBSE OD 1
                                                                                                             Solution. Only the
                                                                                                                                   charge -20 is enclosed
                                                                                                                                                                 by the
                                    on                                                                  sphere of radius 3a. By Gauss's
                                                                                                                                           theorem.
         .Whenthe distance                                                                                                                            20
                                                 of   the point charge         is
  the force reduces to                                                              doubled,
                                      F/8
     Problem 51 Consider the                                                                                      Problem    56.   A point charge +Qis placed at
                              situation shownin Fig,                                                                                                                                 the centre
  What are the signs ofq,                            1.114.                                             O   of an uncharged
                          andg,2 If the lines are drawnin                                                                   hollow spherical
                                                                                                                                              conductor of inner
                                                                                                        radius 'a' and outer
  proportion to the charge,                                                                                                  radius "b. Find the
                                                                                                                                                 foliowing :
  what is        the            4, lq,
                                                                                                          (a) The magnitude
                                                                                                                              and sign of
                       ratio                ?
                                                                                                              the charge induced on
                                                                                                                                       the
         Solution.   is a    Here q                                                                           inner and outer surfaces of
 negative charge and q, is                                                                                    the conducting shell.
 a positive charge.
                                                                                                          (b) The magnitude of
                                                                                                                                   electric
                                                                                                              field vector at a distance
         92       18                                                                                              ()r Gand               (i) r    =   25,
                                                                                                                           ;
                                                                                                                                   at all points on this surface
  electric field in a particular
                                  direction    is                     F. What will be                                                                                              and    will
                                                                                                   Point radially           outwards.
 the magnitude and
                       direction  of the force experienced
 proton at fhe same point in the field
                                                             bya                                                  Flux through the Gaussian
                                                                                                                                                                 surface,
                                             ? Mass of the proton
is 1836 times the mass of the
                                   electron.                                                                                  = Ex4n2
                                                       (CBSE FO7)
     Solution.A proton has charge equal                                                                   Charge enclosed by the
that of an electron. Hence
                                                  and opposite to
                                 the proton will experience a
                                                                                                                                 Gaussian surface                            =+Q.
                                                                                                          By Gauss's theorem,
force equal and opposite to that of
                                          F.
Fig. 1.115 1 1
Problem 1 Compare electrostatic and gravitational P'roblem 4. Two point eiectric charges of unknown
interactions.
                                                                                                          magnitude           and sign are placed a distance                              d' apart. The
       Solution.          Similarities              between            electrostatic          and         electricfield intensity is zero at a point,not between the
gravitational            interactions :                                                                   charges but on the line joining them. Write the essential
              1.   Both forces act according                                                              conditions for this to happen.
                                                                 to     similar laws        :
                                        1
                           F=                                                                                                          The two charges must have opposite signs.
                                                         and       F=G",2My                                      Solution.        (i)
              2.   Electrostatic             interactions       depend on the nature                      Draw the electric                   lines of a point chargeQ                    when    ()Q>0
                    of   the medium while                   gravitational         interactions            and     (ii)   Q   <0.                                                             [CBSE       D 071
                   do not depend on the nature                           of the medium.
                                                                                                                                  The          lim     indicates        that   the test charge q is
                                                                                                                 Solution.
              3.   Electrostatic              interactions         are     much          stronger                                             q0
                   than        gravitational         interactions.
                                                                                                          small enough                  so that        its   presence does not                affect        the
   Problem 2.Distinguish                            betweenelectric               charge      and          distribution   source charge and hence does not change
                                                                                                                                  of
                                                                          distribution               of
                               Represent            the surface
         Problem          3.
              According to Coulomb's
       Ois
                                                   law, force exerted by                q on              Importance :
                                                                                                                  (0        The tangent                any
                                                                                                                                                  at           point         onthe
                                                                                                                        direction         of the                                     Curvegives the
                                                                                                                                                       electric         field   at that point
                                                                                                                  i) The         relative
                                                                                                                                                  closeness of the
                                                                                                                        indicates the
                                                                                                                                                                                     lines   of   fo
             Force exerted       by e, on Qis                                                                                         relative                     strength of         electric fela
                                                                                                                        atdifferent         points
                           1                                                                                 Probleo           What ismeant by the
                                                                                                                                                   statementthat s
                                                                                                    electric     field   of a point
                                                                                                                                     charge has spherical
                                                                                                   whereas           that of an                           symmet
                                                                                                                                   electric dipole is cylindricall
           Acconding to the principle                                                              symmetric            ?
                                                   of superposition,
       exerted by                                                             total force
                    and q, on Qis
                                                                                                        Goltion             The electric          field    due          to   a point charge         a
                                                                                                   distance          r fromit is given                 by:     E
Fig. 1.119
experiences a torque                 due to      which       it    rotates        about an      axis                           Tmin           P   Esin      (0°    or 180°)      =0
nerpendicular to the                 electric     field     and passing through                   its
                                                                                                             Problem             13.     Two        small         identical       electrical          dipoles
mid-point.                                                                                                                                                                        p     are kept at an
                                                                                                        AB        and CD, each of dipole                           moment
       (in) In     anon-uniform            electric         field,      an       electric    dipole
                                                                                forces at    its two    angle       of    120°     to        each other in an             external         electric     fieid
experiencestwo unequal and non-parallel
ends. The two forces add up to give                                      a      resultant force         E pointing             along the x-axis as                   shown         in   Fig.   1.122.
                                                                        T       andE                                                                                                                            X
                 1.7 and       p                                  2.
                                                                                                                 Given Pa         Pe=P
                                                                                                                 Resultant dipole moment,
                                                                                                                                                                                           Fig. 1.123
                                                                                       +4                          PR      =NP          p+2xpxp cos 120°
                     (a)
                                                                                                                 (i) Torque,                  r       pEsin 30°=pE
                                                                                                                                  Z-direction.
                                                                                                         paper along
                                                                                                                                                                            flux.      Write      SI units.
                                                                                                                                                                                                its
                                                                                                                  Problem         14. (a)      Define        electric
                                                                                                                                                                                                 [CBSE 18C]
   Solution.            (a)   The      electric          flux      through      a    given surface                        Solution.          ()For           0<x<a,         the
                                                                                                                                                                                  charge
area    is   the    total     number of                 electric     lines     of    force passing                  Gaussian surface                  Iis    zero.
                                                                                                                                                                                            enclosed           by
It is given by
                                                                                                           it
 surface remains unchanged.
 placed at          its   centre       C and ano ther charge +20                            is   placed                 By Gauss's theorem,
  outside the             shell        at a distance                 x from the centre as
  shown in Fig.               1.124.
   centre
            Solution.
                of the
                              Net   force
                              shell is zero.
                                                  on the charge Q/2, placed                       at the
                                                                                                                                  Ex4ry2          9+Q                      E=     1 q+2
     Force on the charge 20 kept at point A                                            at distance              r
                                                                                                                        Problem        A spherical rubber balloon carries a
                                                                                                                                         17.
  from the centre is                                                                                                charge that     uniformly distributed
                                                                                                                                       is                 over its surface. As
                     F=Ex20=
                                              4ren
                                                  1
                                                         3Q/2 x2Q=
                                                              2                     130                             the balloon is blown up; how does E
                                                                                                                                                             vary for points
                                                                                                                    () inside the balloon,(i) on the surface of the
                                                                                                                                                                       balloon
                                                                                                                    and (iii) outside the balloon ?
            (i) Electric        fiux    through the                shell,        =    Q2                                Solution.        (i) For      points inside        the balloon,     E   =   0.
                                                                                                                        (ii)    As the      balloon     is   blown up,      surface    charge density
            Problem       16.    Two         thin        concentric          and coplanar sphe-                     G decreases         and     so     the field,     (E=o/E)          on its surface
  rical shells,           of radii a and                b (b>a)carry           charges,        q and       Q.       decreases.
             Problemn         1.The     dimensionsof                 anatomn        are of the order                    Solution. The                   electric      fields      of    protons          and
   of   an      Angstrom.           Thus          there       must be        large    electric     fields           electrons bind the atoms                      to a neutral    entity.   Fields       are
     between        the       protons         and electrons.  Why,                     there      is    the         caused by excess charges, there can be no excess
   electrostatic field inside                 a conductor zero                                                                                                       charg
                                                                                                                    on the inner surface of an isolated conductor.
                                                                             [ExemplarProblem)
                                                                               ELECTRIC           CHARGES AND                FIELD
         Problem        2.   Twodipoles,
                                made from charges tqand                                            +O
esDectively.  kave equal dipole moments. Give the
                                                        (i) ratio
letpeen the 'separations' of these
() qa =Qd
                                                                                       ++   ++                                       (a)
                                                  must have
                                                                                  +
       (ii) their      dipole axes
same direction                i.e.,   0=0°
       Problem 3.Sketch                   the electric field
(C
(a) (e)
               ()
                Top view                                               (ii)   Side view
                                                                                                                                                      Fig. 1.127
       Problem 4. Two point charges placed at a distance r in                                                  Figure           1.127(a)     is     wrong because       field     lines     must be
           a force F on each ofher. At what distance
       exert                                         will these                                            normal      to    a conductor.
 air
charges     experience          the       same     force   F in    a   medium of dielectric                    Figure            1.127 (b)   is   wrong because        lines     of   force   cannot
constantK?                                                                                                 start   from a negative charge.
                         med          =
                                              1
                                                       9192   =F                                  ..(ii)   intersecteach other.
                                                                                                          C.
 the neutral                                                                                                                                                                                                                               y-direction,
                       point will              be closer to the charge with smaller                                           Force on thecharge-                      q   in   the upward               direction            is
 magnitude. Hence,                         electric                                                                                                                                                                                            (a)     Negative           cha
                                                          field is zero            near charge A.
                                                                                                                                                                 ma = qE                                                                       (b)     (ii)    and      (iv).
        Problem             8.   A glass          rod    rubbed with             silk           brought close
                                                                                        is
                                                                                                                                                                           qE
                                                                                                                                                                                                                                                       same sign ar
 to   fwo     uncharged           spheres          in contact        with    each       other, inducing
                                                                                                                              :. Acceleration,                   a=
 charges             on     thenm           as
                                                                                                                                                                                                                                                (c)    To find e/
                                                                                                                                                                                                                                                       ment h as the
 shown          in     Fig.       1.129.                                                                                      Time taken            to   cross the field,              t   =
 Describe            what happen                                                                                                                                                                                                                 Time taken by a
 when
                                                                                                                              Vertical                         at the far edge of the plate
                                                                                                                                             deflection
                                                                                                                                                                                                                         will       be                                  t=
        (i)     the       spheres           are     slightly                                                                                         1
                separated,            and                                  Fig. 1.129                                                 y=    ut   + at?
                                                                                                                                                   2   =0+
                                                                                                                                                           2                                                                                     Problem            11.   Fi
                                                                                                                                                                                  1m           U
                                                                                                                                                                                                    2mu
      (i) the          glass         rod    is subsequently                                                                                                                                                                                  of electric       field    line
                                                                           removed, and               finally
                                                                                                                              Like the motion of a                projectile               in gravitational                                  from     rest     at pointP
      (iii) the spheres                are separated                                                                                                                                                                          field,
                                                                far apart           ?                                     the path of a charged                   particle                 in an       electric          field       is      by the        electric field.
        Solution.
                                                                                                                          parabolic.
                                                                                                                                                                                                                                             momentum              of   the
                           A               B             A             B                A                      B
                                                                                                                              Problem 10. Figure 1.132 shows -tracks of
                                                                                                                                                                            three                                        charged
                                                                                                                          particlesin a uniform
                                                                                                                                                electrostatic field. Give the                                      signs      of    the
                                                                                                                          three   charges.       Which      particle       has the highest                  charge       to    mass
                                                                                                                          ratio   ?
                                                                                                                                                                                            [NCERT;CBSE D 01C]
                                 (a)                           (6)                                  (c)
Fig. 1.130
            h=0                 2 a²19E2
                             xt+-   2 m                                                                        straight    line for both pairs of charges.
     As         the particle
                                                                                                                    ()For the  pair (1 uC, 2uC), Fis repulsive,the graph
                                           suffers          maximum              defection          in         OA    has a +ve slope. For the pair (2uC,
 y-direction,
                        so    ) highest charge
                             ithas
                                                                   to mass (g/ m)
                                                                                                                                                             -3uC),F is
                                                                                 ratio.                         attractive, the graph OB has     -ve slope.     a
                                                :.     h=E2
                                                        2 m -0.5
      Problem 11. Figure 1.133 shous                              three      different     patterns
of   electric field           lines.    In each        pattern,        a   proton        is released
 from      rest at   point P and           then      accelerated        towards the         point     Q
 by the     electric field.       Rank the patterns according to the                            linear
momentum                     the proton when it reaches Qgreatest
                        of
                                                                                                first.                                              Fig. 1.134
                                                                                                                                                               19
                                                      (b)
                                                                                                                                                          AY
P 92 (0, b, 0)
(c)
  COulomb
                                                       1
vetoeen thetuo charges of each pair of charges :(1 uC, 2uC) Enet = 14-ai+ ck).-bj- cky
      Problem 14.                      A peint        datge         is   pland ot thecentre                of    6elds             and             are equal            and      opposite          and          also    E
splherical       Giussian               surteHoe oilleiectri fuxh chnge                                                                                                                                          feld    a
                                                                                                                        E, are equal and opposite. Hence
                                                                                                                                                         the net
                                                                                                                 and
                                                                          by an                            ?           Problem
     (i)      the oiginal charge is replaced                                          electric dipole
                                                                                                                                                                                                              the dipole
                                                                                                                                                                           triangle.     What           is
                                                                                                                      the verices of an equilateral
                                                                             -E.aš - I                           at
       Solution                By         Gauss's law,                                                            moment of the system
                                                                                                                                                      ?
                                                                                                                                                               24     pC
                                                                                                so the net
                                                                                                            of
                                       field     at   the centre O.In                Fig.    1.137i), the        hei      eachi     thread                      small
       there     is   a   net                                                                                                                                                                JEE Main            July       21]
                                                                                                                                            F=
                                                                                                                                                      1
                                            ()                                     (i)                                  For    smallangular         displacement                        0,   we      have
                                                                                                                                                           arc                           x/2             x
                                                                                                                                      sin    0-0(rad)
                                                       Fig. 1.137                                                                                        radius                               1         21
                                                                    ELECTRIC                   CHARGES AND FELD                                                                                            L75
                                                                                                             Problem 19. A                  simple pendulum                         consists          of    a   small
                                                                g
                                                                                                             Acceleration,
                                                                                                                                                                                               17
      .Restoringforce on                    each    ball
                          =   1Ig   sin      -
                                                                                                                                   T=2n-2r
                                                                                                                                                                                    m
      For equilibrium,
                                                                                                             Problem 20. Four                  particles,              each having a charge q, are
                                1
                                                                                                      placed     on   the vertices            of a regular              pentagon. The distance of
                                                                                                               corner    fromthe           centre           is 'a'.    Deternnine             the electric field
                                                                                                      each
 Forces
                                                                                                       field    due     to    the charge                q at     E
equal and opposite and
                       hence cancel out.
                                                                                                       alone.
                                                                                                       charge q at           E
                                                                                                                                              1              4
                                                                                                                                                                      along     EO
                                                                    C
along OE.
                                                           |                                                   ()Find
        Similarly,
                                                                                                                              the electricfield
cancel out.
                                                                                                 by
                                                                    O   is exerted                                 up ofchages.
        The                      on charge              -4 at                                                                                                             above set up,
        only force                                                                                                                       be the electric field, if in the
                                                                                                             (i) What          will
charge +q at C. It is           given by                                                                                                               charges have             opposite            signs.
                                                                                                                   the consecutive
                               kq newion,along                                           OC.                                                                                                                    [IIT 95]
                        F=k194
                                 (Oc)
  176
                                                                                                  PHYSICS-XI
2F
                                                                                    1
                                                                                                                          Fpet                    cos       9
                                                                     AnE,      1
                                                                                                                                                2kQgx
          (ii)
             Electric       field     at    a       =0                                                                                                              3/2
                                                         is
                                                                                                                                                       4                                            d/2              d2
                                      42
                                                                                                                      For      maximum                  value             of         t               Fig. 1.143
                                                                                                                                 dFnet
                                                                                1
                                                                                                                                                     =0
                          4      16        64                                                                                             dx
                                                                                                                                                                                                          1/2
          Problem 22. Two              identical              positive    charges       Qeach are                                                                                                               x2r=0
 fixed      at a distance        of   2a'apart                 from each      other.      Another
 point charge          g,with mass 'm'is placed                          atmidpoint between                                                                 d
 fuofixed           charges.     For a      small         displacement         along       the line
 joining      the fixed carges, the charge                                                                                                             2/2
                                                               q, execiutes   SHM.Find the
  time period         of oscillation                                                                         Problem 24. Eight identical point charges of q
                                            of       charge go:             JEE Main       July   22]     each are placed at the                               coulomb
                                                                                                                                 corners of a cube of each side 0.1
          Solution.                                                                                                                                                 m.
                                                                                                          Calculate                 the electric field at the centre  of the cube.                        G
                                                                                                          Calculate                 the field at the centre when one
                                                    m                                                                                                                of the corner
                                                                                                         charges              is    removed.
                                                                      a-x                                            Solution.Length of each side, I =0.1                                             m
                                                                                                                     Distanceof each corner from the
                                           Fig. 1.142                                                                                                                                      centre of the cube                 is
                                                     x=-kx
                                                                                                                                                       9x 10 x q
                                                                                                                                                                                         =1.2   x               g NC
                                                                                                                                                     (5/3x            102                            10
we get,        q=0
ie. no net charge is enclosed by                               the Gaussian             surface.
                                                                                                                                                           Fig. 1.145
Problem 26. Figure 1.144 shows a cylindrical Gaussian Solution.(i) To determine the electric field at point
surface jor an infinitely long thin straight eire of uniform P, consider a concentric spherical surface of radius r,
linear    charge density.                                                                                      as the Gaussian surface.                    By symmetry, the field E will
                                                                                                               have same magnitude                       at all points on this surface and
                                                                                                                                             = Ex 4r
                                                                                                                      Charge enclosed                 by Gaussian           surface =      +Q
                                                                                                                      By Gauss's            theorem,
                                                                                                                                                                                                 1
                                                                                                                              =                     Ex   4ri                   or         E=
      )
                                               Fig.    1.144
                                                                                                                      (i) As the electric              field    insidea conductor               is   zero, so
      ()
             For which surface is the
                                                              zero     ?
                                                                           flux zero      ?
                                                                                                                radius
                                                                                                                placed
                                                                                                                       Problem 28.
                                                                                                                              r,   and outer
                                                                                                                             at the centre
                                                                                                                                                  A
                                                                                                                                                  radiuS
                                                                                                                                                   of the
                                                                                                                                                             ,
                                                                                                                                                       splherical        conducting
                                                                                                                                                                 has a charge
                                                                                                                                                               shell.
                                                                                                                                                                                          shell
                                                                                                                                                                                      Q.A charge g
                                                                                                                                                                                                     of inner
                                                                                                                                                                                                     ()inner
                                                                                                                                                                                                              is
(ii) Over which surface is| E| constant ? (a) What is the surface charge density on the
                                                                                                                              Write
                                                                                                                                             (i)outer surface
                                                                                                                                           the   expression
                                                                                                                                                                         of the   shell    ?
4-[E.aš =0.
     (ü)
               E
               At
                   cannot
                     all
                                be
                            points
                                         zero.
                                               of the curved
                                                                                 surface,       |    E|   is
Constant.
             increase in distance                       r.
                                                                                                                                     PHYSICS-XII
           L78                                                                                                                                                                    1.148 shows the
                                                                                                                                                     Solution.        Figure
                                                                                                       inner surface,                                                                               charge
                            charge                          density                   on the                                                                                     model of the atom.                            distr
charge.
                                                            Q+
                                  E=             1
Problem 29. Electric field in Fig. 1.146 is directed along Fig. 1.148 An early model of atom.
                                                                                                                                                              ;=
                                                                                                                                                                                                                  surface,
          The magnitude                      of the              electric                  field           at the face
   M(I =0)is                                                                                                                                                          Ex 4n?
            E =5 Ax +2 B=5x 10                                        x         +2x5=10 NC-1
                                                                                0                                                             Charge enclosed               by the Gaussian          surface,
                                                                                                                                                             E)
        Flux,                                    cos                                                                                                               Ze
                                                                                                                                                   Ex
                                =15 x (0.10)x                         cos       0°=0.15 Nm                      2c1
        Net flux          through the cube,
                                                                                                                                        or
                    =       t y =-0.1+0.15                                  =       0.05        Nm ?c1.                                                                                                                (r<R)
        (ii)Total         charge enclosed within the cube,                                                                                   The   field     Epoints radially
                                                                                                                                                                                      outward.
             4=e                =8.854        x   10x0.05                             =4.43            x 10-13                               (i) For       r>R. As           the   atom    is    neutral,        the total
                                                                                                                           C.
    Problem 30. An
                                                                                                                                       charge enclosed            by       the Gaussian surface             is   zero.     By
                                      early      model ofan atom
 haoe npositiely
                                                                                                considered            it        to
                                                                                                                                        ualiis    s theorem,
                           charged         point nucleus                            charge Ze, surrounded
                                                                          of
byauniform              density       of   negative         charge              up toa               radius     R.    The                                      Ex 4r=0
atom as         a   whole is           neutral.        For        this              model,            what       is    the                                         E- 0.                                            (r>R)
electric                ata distance          r from the nucleus
            field
                                                                                    ?                                                        (i)Atr=         R. Both of the above cases give                     the    same
                                                                                [NCERT;AlIMS 18)                                      result:      E=0.
                                                                                                 ELECTRIC CHARGES                    AND         FIELD                                                                                              79
                                         2x
                                                                                                 charged       spheres
                                                                                                                                    The      factor               ke| Gmn,                             represents                 the      ratio     of
  kaing charges                     of         T0          Cand 3 x 10Cplaced 30cm                               apart        clectrostatic         force               to the              gravitational               force betweer                an
       air
 in
                                                                                                                             electron       and a protor. Also, the large value                                                       of   the ratio
        Ans.         Here           4 =2x 10                     C,   q,    =3x107C.                                         signifies       that       the electrostatic                         force is           much stronger thar
                                     r=30cm                 =0.30m                                                           the    gravitational                 force.
                                                                                                                                                                   of
                                                                                            x3x107
                              2T2-9x10,2x
                                1
                 F=                                                                  10
                                                                                                                                           (i)   Why          can one              ignore         quantisation                of electric       charge
                                                                                       (0.30)                                                    when             dealing           with macroscopic                           ie.,     large      scale
                4,   = -0.8uC=-0.8 x 10                                     °C        F=0.2N, r=?                            because e is very small and nis very large                                                   andsoq behaves
                                1                                                                                            as    if it   were continuous                         i.e.,         as   if    a large      amount of charge
       As       F=                           2                                                                               is   flowing continuously.
                                                                                                                                    1.5.   When a glass                 rod   is   rubbed             with    a silk    cloth, churges          appear
                                1
                                             492                                                                             on both.       A    similar             phenomenon                        is   observed          with many            other
                                                                                                       value     of   this
4p     atable                 physical        constants and determine                           the
                                                                                                                             of the    sqare        ?
                     of
ratio.      What          does this ratio                 signify       ?
                                                                                                                                    Ans, Here            OA =OB=                         0C= OD=                               2
                                                            [Nm?c                    x[C?             = nounit
       Ans.               k
                                                                                                                                                                        =5/2cm=5/2 x                                 10m
                              Gmm, [Nm'kg 1 *(kgl[kgl                                                                                                                                       10em                 4-2
                                                                                                                                                         4o-5HC                                                           C
dimensionless.
       Now                      k=9x 10 Nm'c2
                                G=           6.67 x    101 Nm'kg
                                    e=       1.6    x 101 kg
                                =9.1x 10
                                m,                               kg
and
                                     =
                              m, 1.66 x 10                            kg
                                                          9x     10         x(1.6     x 10       19                                                                                                                           B
                                                                                                                                                                                            10   cm
                                                                                                        x 10    27
                                                                                 103        x   1.66
                                                     x10          x9.1 x
                     Gm         y            6.67
Fig. 1.149
                                         =   2.287        x10*
                                                                                                  PHYSICS-X1
                                                  }
                                              (52×10                                                                                 Eg
                                                                                                                                                                               9x10           3        x105
                                               (5V2 10
                                                                                                               (i) Force on a
                                                                                                           the midpoint O,
                                                                                                                              negative charge of                                            1.5 x     10"C placad            .
                                -9N,along OD                                                                                           F= qE= 1.5 x 10                              x5.4 x       10
                                                                                                                                                                                           the    total
                                                                                                                                                                                                            -15om)     and
                                                                                                                                                                                                                           in
         Ans. (a)
                                                                                                 OD14]
                  Electric lines of                                                                                                  2a    =30cm
region      of an
                   electric field.
                                                        force
                                                  throughout the
                                The eleciric field of a
                                                                  exist                                                                                            =   0.30     m, q      =2.5 x       10C
                                                            charge
decreasesgradually with
                             increasing
                                                                                                                                                                       Z
becomes zero at infinity                   distance   from it and
                              Le.,electric
                                             field cannot
abruptiy.Soa line of force                                  vanish                                                                 4,=-2.5 x             10       C
                                cannot have sudden                                                                                                                        B (0, 0, + 15 cm)
must be a continuous                                     breaks, it
                         Curve.
    (b) Iftwo lines of
                        force intersect, then
two tangents and hence two                        there would be
                                  directions   of electric field at
the point of intersection,
                             which is not possible.
       1.8.    Two point         charges            4, =+3uC and               q       =-3 uC       are
located       20cm apart         in    vacuum.         () Find     the                           at the
midpoint          O of    the line           ABjoining      the tuo
                                                                           electric      field
negative
                                                                              charges.          (i) 1f a
                test   charge               magnitude     15 x 10             C                                                                                           A0,0, - 15 cm)
                                  of
                                                                                  is   placed    at the                         4-2.5           x       1o'c
centre,    fnd the force          experienced            by the    test    charge
                                                                          (CBSEOD 031
         Ans.     Thedirections              of the     fields    E, and E, due to the                                                                    Fig. 1.151
  cCharges        4    and 4           at    the mnidpoint         P are as shown in
 Fig      1.150.
                                                                                                                  Total       charge       =4 t               q    =2.5 x 10-2.5x10                             =0
                                                                                                                Dipole moment,
                            10 cm                                                                                                    p=qx2a                                    10
                                                                   10   can                                                                                       = 2.5    x          x0.30
                                                                                            B
                                                                                                                                       -0.75              x   10 Cm
                                                                                                               The dipole moment acts                                  in the direction                from     B to   A
                                               Fig.1.150
                                                                                                           Le, along negative Z-axis.
                                                                                      ELECTRIC                     CHARGES AND FIELD                                                                                                                    L81
       L10.        Anelectric dipole                with dipole           moment 4          x        10           Cmis                  1.14. Consider                    a uniform                   electric      feld :
              at    30 oith the                 direction       of   a uniform                                                                                                                                                                      through
 aligned                                                                                  electric            ficld                       3   x     10              NC             )Wihat is the                   flux        of   this   field                   a
                                                                                                                          of
                                                                                                                                                           i
                                                                                                of
                                                                                                                                               of                                      a
                                                                                                                                 sgquare
            on the      dipole.
acting                                                                                                                           Y-Z-plane               ?(ii)      What          is       the flur      through the same square if the
           Ans.     Here      p=4x10                    Cm,9-         30°,       E   =5x 10' Nc!                                 normal to          its   plane makes a                          60      angle with the X-axis ?
                                                                                                                                                                                                                          (CBSE D 13C]
           .Torque,          t=pEsin                e
                                                                                                                                                                                                                                     Y-Z plane points
                                =4x10x5x10' xsin                                30°
                                                                                                                                       Ans. ()Normal
                                                                                                                                 in X-direction,                    so
                                                                                                                                                                                 to a       plane           parallel           to
                                = 10 Nm.                                                                                                                                                                                                        m?
                                                                                                                                                           AS = 0.10 x0.10í m² =                                                  0.01
1.11. A polythene piece rubbed with wool is found to have a Electric flux,
polythene
                   ?                                                                                                                                                    -30í.í =                        30    Nm²c.
           Ans.()Here                 =3.2x          10     C, e=1.6x10-19                            C
                                  q
transferred
                                       has negative charge, so
                        from wool to polythene during rubbing.
                                                                                           electrons                 are
                                                                                                                                    of side 20
                                                                                                                                 cube
                                                                                                                                 coordinate
                                                                                                                                               oriented so that
                                                                                                                                                     planes
                                                                                                                                                                   c
                                                                                                                                                                fuces are parallel to the
                                                                                                                                                                        ?
                                                                                                                                                                                                 is
ifs
                                = x 10 kg
                                      1.82                                                                                       Surface       ofthe box             is     .0 x           10°        NmC.()What                           is    the net charge
small.
      Clearly,
1.12. (a)
 mutual       force
                        the
                        Two
                               amount of mass transferred
insulated
separated
                        of electrostatic repulsion
                                                     charged copper spheres A and B
                                                    by a distance of 50 cm. What
                                                   if
                                                                                 is the
                                                                                            is negligibly
inside
                                                                                                                                        Ans.        ()    ,
                                                                                                                                       box were zero, could you conclude
                                                                                                                                         the box ? Why or why not ?
                                                                                                                       to
65     x   10C?          The radii of A and Bare negligible                                     compared
                                                                                           of repulsion                     if
the distance                 separation.           (b)What           is   the force
                                                                                                                                 edge 10 cL)
                                                                                                                                                                                                        with
                                                                                                                                           Ans.      We        can imagine the square as face of a cube
                                                          1.152                                                                  edge         10    cm         and with the charge of + 10uC placed at its
                                                  Fig.
                                     (a)
                                                                                     (h)                                                 1.21.      A uniformly charged
                                                                                                                                                                                                     conducting                  sphere
                                                          Fig. 1.153                                                              diameter       has    a surface          charge density                       of      80.0     uC/
                                                                                                                                  the    charge on the sphere.                  (ii)             What          is   the      total
           Symmetry of                        six    faces            of a cube about                                 centre                                                                                                           dlectric    f
    ensures
                                                                                                            its                  leaving        the surface       of the sphere                  ?
                       that       the    flux         through each square                                                                                                                                                             CSED
    when the                                                                                           face       is   same                                 R=      2.4
                           charge q is              placed at the centre.                                                               Ans. Here                          = 1.2 m
                 Total
                                                                                                                                                                     2
                                flux,
1.18.
    Gaussian
                            A
                        surface
                                point    charge
                                        9.0   cm on
                                                         of 2.0
                                                           edge.
                                                                      uC
                                                                        What
                                                                            is   at the centre
                                          2.0    × 10-6
                                                                                     10°   Nm2         c-1
                                                                                                                                      Ans.      E=9x10*             NC, r=2cm =0.02                                          m
                                        8.85     x 10-l2               2.26      x
  Gaussian
                                                                                 (i) If the           radiusof the                                                                                              x9x10* x0.02
                       surface          were     doubled,              how much                                                                                                4n x9x109
                                                                                               flux    would pass
  through the                            ?(i1) What
                           surface                              is   the value        of the point          charge       ?
                                                                                                                                                =0.01 x 10             Cm            =0.1            uCm.
     Ans.               (i)       r=-10                  NmC,                    because              the                             1.23. Two large,
 enclosed             is   the same in both the                                                               charge                                           thin       metal      plates           are parallel and close
                                                                          cases.                                                                                                                                                                    to
                                                                                                                               each   other. On their
                                                                                                                                                               inner faces, the
       (i) Charge,                                                                                                                                                              plates
                                                                                                                                                                            have surface                                                    charge
                                                                                                                               densities of opposite signs and of
                                                                                                                               What is E (a) to the               magnitude 17.0 x                                               102        C
                                                                                                                                                             left   of the plates,               (b) to        the right of the plates.
                                                                                                                               and    (c) between       the plates         ?
                                                     1
-19.2 x1010 NC l